SlideShare a Scribd company logo
TO GET ALL CHAPTERS EMAIL ME AT>>>>> donc8246@gmail.com
Test bank For Wilkins clinical assessment in
respiratory care 8th edition by Huber,
Chapters 1 - 21
TO GET ALL CHAPTERS EMAIL ME AT>>>>>
donc8246@gmail.com
Wilkins' Clinical Assessment in Respiratory Care, 7th Edition
Contents:
Chapter 1. Preparing for the Patient Encounter
Chapter 2. The Medical History and the Interview
Chapter 3. Cardiopulmonary Symptoms
Chapter 4. Vital Signs
Chapter 5. Fundamentals of Physical Examination
Chapter 6. Neurologic Assessment
Chapter 7. Clinical Laboratory Studies
Chapter 8. Interpretation of Blood Gases
Chapter 9. Pulmonary Function Testing
Chapter 10. Chest Imaging
Chapter 11. Electrocardiography
Chapter 12. Neonatal and Pediatric Assessment
Chapter 13. Older Patient Assessment
Chapter 14. Monitoring in Critical Care
Chapter 15. Vascular Pressure Monitoring
Chapter 16. Cardiac Output Measurement
Chapter 17. Bronchoscopy
Chapter 18. Nutritional Assessment
Chapter 19. Sleep and Breathing Assessment
Chapter 20. Home Care Patient Assessment
Chapter 21. Documentation
TO GET ALL CHAPTERS EMAIL ME AT>>>>>
donc8246@gmail.com
Chapter 1: Preparing for the Patient Encounter Test
Bank
MULTIPLE CHOICE
1. Which of the following activities is not part of the role of respiratory therapists (RTs) in
patient assessment?
a. Assist the physician with diagnostic reasoning skills.
b. Help the physician select appropriate pulmonary function tests.
c. Interpret arterial blood gas values and suggest mechanical ventilation changes.
d. Document the patient diagnosis in the patient’s chart.
ANSWER: D
RTs are not qualified to make an official diagnosis. This is the role of the attending physician.
REF: Table 1-1, pg. 4 OBJ: 9
2. In which of the following stages of patient–clinician interaction is the review of physician
orders carried out?
a. Treatment stage
b. Introductory stage
c. Preinteraction stage
d. Initial assessment stage
ANSWER: C
Physician orders should be reviewed in the patient’s chart before the physician sees the
patient.
REF: Table 1-1, pg. 4 OBJ: 9
3. In which stage of patient–clinician interaction is the patient identification bracelet checked?
a. Introductory stage
b. Preinteraction stage
c. Initial assessment stage
d. Treatment stage
ANSWER: A
The patient ID bracelet must be checked before moving forward with assessment and
treatment.
REF: Table 1-1, pg. 4 OBJ: 9
4. What should be done just before the patient’s ID bracelet is checked?
a. Check the patient’s SpO2.
b. Ask the patient for permission.
c. Check the chart for vital signs.
d. Listen to breath sounds.
ANSWER: B
It is considered polite to ask the patient for permission before touching and reading his or her ID
bracelet.
TO GET ALL CHAPTERS EMAIL ME AT>>>>>
donc8246@gmail.com
REF: pg. 3 OBJ: 3 | 5
5. What is the goal of the introductory phase?
a. Assess the patient’s apparent age.
b. Identify the patient’s family history.
c. Determine the patient’s diagnosis.
d. Establish a rapport with the patient.
ANSWER: D
The introductory phase is all about getting to know the patient and establishing a rapport with
him or her.
REF: Table 1-1, pg. 4 OBJ: 3
6. Which of the following behaviors is not consistent with resistive behavior of a patient?
a. Crossed arms
b. Minimal eye contact
c. Brief answers to questions
d. Asking the purpose of the treatment
ANSWER: D
If a patient asks about the purpose of the treatment you are about to give, this generally indicates
that he or she is not upset.
REF: Table 1-1, pg. 4 OBJ: 3
7. What is the main purpose of the initial assessment stage?
a. To identify any allergies to medications
b. To document the patient’s smoking history
c. To personally get to know the patient better
d. To verify that the prescribed treatment is still needed and appropriate
ANSWER: D
When you first see the patient, you are encouraged to perform a brief assessment to make sure
the treatment order by the physician is still appropriate. The patient’s status may have changed
abruptly recently.
REF: Table 1-1, pg. 4 OBJ: 3
8. What is the appropriate distance for the social space from the patient?
a. 3 to 5 feet
b. 4 to 12 feet
c. 6 to 18 feet
d. 8 to 20 feet
ANSWER: B
The social space is 4 to 12 feet.
REF: pg. 5 OBJ: 5
9. What is the appropriate distance for the personal space?
TO GET ALL CHAPTERS EMAIL ME AT>>>>>
donc8246@gmail.com
a. 0 to 18 inches
b. 18 inches to 4 feet
c. 4 to 12 feet
d. 6 to 15 feet
ANSWER: B
The personal space is about 2 to 4 feet from the patient.
REF: pg. 5 OBJ: 5
10. Which of the following activities is best performed in the personal space?
a. The interview
b. The introduction
c. The physical examination
d. Listening for breath sounds
ANSWER: A
The interview is best performed with you sitting about 2 to 4 feet from the patient. If you sit
farther away, the patient will have to answer your questions in a louder voice, and because some
of the information may be private, this would diminish communication.
REF: Table 1-1, pg. 4 OBJ: 5
11. What type of behavior is least appropriate in the patient’s intimate space?
a. Eye contact
b. Pulse check
c. Auscultation
d. Simple commands
ANSWER: A
Eye contact is inappropriate in the intimate space and will make the patient very uncomfortable.
REF: pg. 3 OBJ: 5
12. You are riding in an elevator at the hospital where you are employed as an RT. The elevator is
full, but standing next to you is Joe, the RT who is scheduled to relieve you. He turns to you
and asks, “How is Mr. Copper doing?” Earlier in the day, Mr. Copper had a cardiac arrest, and he
is now being mechanically ventilated. How should you respond to Joe?
a. “He took a turn for the worse.”
b. “He is fine.”
c. “Let’s talk later in the report room.”
d. “He is on a ventilator and will keep you very busy.”
ANSWER: C
The patient’s right to privacy prevents care providers from discussing a patient’s clinical status in
public places. All answers other than “c” are unethical; giving such answers could cause an RT to
be in legal trouble and get fired.
REF: Table 1-2, pgs. 4-5 OBJ: 6
13. In 1996, Congress passed the HIPAA. What does the letter “P” stand for?
TO GET ALL CHAPTERS EMAIL ME AT>>>>>
donc8246@gmail.com
a. Patient
b. Payment
c. Portability
d. Personal
ANSWER: C
HIPAA stands for Health Insurance Portability and Accountability Act.
REF: pg. 4 OBJ: 6
14. Which of the following techniques for expressing genuine concern is the most difficult to use
appropriately?
a. Touch
b. Posture
c. Eye contact
d. Proper introductions
ANSWER: A
Touch is most difficult to use properly because gender and cultural differences often become
an issue.
REF: Table 1-1, pgs. 3-4 OBJ: 5 | 7
15. Which of the following techniques is not associated with the demonstration of active
listening?
a. Good eye contact
b. Taking notes while a patient is talking
c. Asking for clarification
d. Use of touch
ANSWER: D
Use of touch helps with demonstrating empathy but has little to do with active listening.
REF: pg. 2 OBJ: 2
16. Two respiratory care students are taking their lunch break and want to compare notes about
patients they have seen during the morning. Which of the following locations would be
considered a violation of HIPAA standards?
a. The unit nursing station in front of the unit clerk’s desk
b. A table in the cafeteria with no one within hearing distance
c. The respiratory department report room
d. The intensive care unit (ICU) staff break room
ANSWER: A
Patient Health Information (PHI) should be discussed only in nonpublic areas of the hospital.
The space in front of the unit clerk’s desk is likely to be occupied with members of the public
asking for information.
REF: Table 1-2, pgs. 4-5 OBJ: 6
TO GET ALL CHAPTERS EMAIL ME AT>>>>>
donc8246@gmail.com
17. A 20-year-old respiratory care student enters the room of a 65-year-old female patient, saying,
“Hi, Linda! I am Joe from Respiratory Care.” He immediately approaches her, looks her in the
eye, and places his stethoscope on her chest.
a. Joe’s approach to this patient is appropriate.
b. Joe has inappropriately entered the patient’s social space.
c. Joe has inappropriately entered the patient’s personal space.
d. Joe has inappropriately entered the patient’s intimate space.
ANSWER: D
Joe has established no rapport with this patient, has touched her without asking permission, and
looks her in the eye while examining her.
REF: pg. 4 OBJ: 7
18. A respiratory care student returns from a clinical experience , excited that she has had the
opportunity to perform cardiopulmonary resuscitation (CPR) for the first time. She immediately
goes to her Facebook page and describes her day. Which of the following entries would be a
violation of HIPAA standards?
a. “At clinical today got to do CPR on a patient on the 6th floor of Mercy Hospital.
Patient survived! What a rush!!”
b. “Got to do CPR for the first time today. Patient survived!! What a rush!”
c. “Got to do CPR for the first time in clinical today! What a rush!!”
d. All of the above
ANSWER: D
Patient Health Information (PHI) must not be shared in a public location. Facebook is
considered a public forum. Although the student did not give specific identifiers in answers a, b,
or c, there was enough information that someone familiar with either the patient or the student
could possibly have deduced the identity of the patient.
REF: Table 1-2, pgs. 4-5 OBJ: 6
19. Which of the following would be the most appropriate way for respiratory care student Amy
Long to initially approach a 58-year-old female patient, Mrs. Nora Jones?
a. “Hello, Mrs. Jones. I am Amy from respiratory care, and with your permission I
would like to assess you for your treatment.” (Amy stands 5 feet from the patient
and makes direct eye contact.)
b. Hey there, Nora! Isn’t this a great day!?! I’m Amy and I need to listen to you.” (Amy
holds out her stethoscope in front of her and approaches the patient to within
1.5 feet.)
c. “Hi, I’m Amy, here to give you your treatment.” (Amy makes no eye contact and
looks around the room for a nebulizer.)
d. Hi, Mrs. Jones. I’m here for your treatment.” (Amy makes direct eye contact.)
ANSWER: A
The initial contact with a patient should be from the patient’s social space (4 to 12 feet).
Patients should be addressed by their last name. When first speaking to a patient, the therapist
should make direct eye contact, but he or she should not use direct eye contact when in the
patient’s intimate space.
REF: Table 1-1, pgs. 3-4 OBJ: 2 | 7
TO GET ALL CHAPTERS EMAIL ME AT>>>>>
donc8246@gmail.com
20. The umbrella term patient-centered care includes all of the following elements except:
a. Individualized care.
b. Assistance with financial and insurance issues.
c. Patient involvement.
d. Provider collaboration.
ANSWER: B
Patient-centered care involves individualized care, patient involvement, and provider
collaboration.
REF: pg. 2 OBJ: 1
21. The golden rule of bedside care can be summarized as:
a. Patients should be cared for primarily at the bedside.
b. All patient services (e.g., x-ray, nursing care, respiratory care) should be delivered to
the patient at the bedside whenever possible.
c. As a caregiver, at all times treat a patient as you would hope to be treated if you
were the patient.
d. Make sure that all safety equipment is in place at the bedside for maximum patient
protection from hazards such as falls.
ANSWER: C
The golden rule is that as a caregiver, you treat patients the way you wish to be treated.
REF: pg. 2 OBJ: 1
22. In interacting with patients, behaviors such as body movements, touch and eye movements,
and facial expressions would be examples of:
a. Nonverbal communication.
b. Expressions of caregiver interest in patient welfare.
c. Mechanisms to put patients at ease.
d. None of the above.
ANSWER: A
These are mechanisms of nonverbal communication that help to put patients at ease and can be
used to communicate caregiver concern to patients.
REF: pg. 2 OBJ: 2
23. In determining the course of treatment for a 20-year-old patient hospitalized for exacerbation
of cystic fibrosis, the most effective course of action would be:
a. Formulating a treatment plan based on the therapist’s knowledge of the disease and
its treatment and then presenting it to the patient.
b. Formulating a treatment plan with the physician and nurse and then presenting it to
the patient.
c. Interviewing the patient and strictly following the patient’s preferences with regard to
treatment.
d. Interviewing the patient to determine his or her preferences for treatment,
formulating a treatment plan in collaboration with the nurse and physician based on
both patient preferences and the team’s knowledge of the disease and its treatment,
TO GET ALL CHAPTERS EMAIL ME AT>>>>>
donc8246@gmail.com
and presenting it to the patient.
ANSWER: D
Patient-centered care must be highly collaborative, with input from both the patient and
caregivers.
REF: pg. 7 OBJ: 4 | 8
24. While interviewing a patient in a room with another patient and that patient’s family present,
the appropriate course of action for an RT would be to:
a. Introduce himself or herself to the patient from a distance of about 5 to 7 feet and
proceed with the interview.
b. Introduce himself or herself to the patient from a distance of about 5 to 7 feet, move
to within 2 to 3 feet of the patient, and proceed with the interview.
c. Introduce himself or herself to the patient from a distance of about 5 to 7 feet, move
to within 2 to 3 feet of the patient, draw the privacy curtain around the bed, and
proceed with the interview.
d. Introduce himself or herself to the patient from a distance of about 5 to 7 feet, draw
the privacy curtain around the bed, sit on the bed about 1 foot from the patient, and
proceed with the interview.
ANSWER: C
Normally, interviews are carried out in the personal space (within 18 inches to 4 feet of the
patient), not the intimate space (from 1 to 18 inches from the patient). The privacy curtain
should be in place because others are in the room.
REF: pg. 3 OBJ: 3
25. A male therapist is discussing a treatment plan with a female patient who is sitting up in bed,
dressed in a hospital gown and wearing a full head covering with only her face showing. Her
husband is in the room, and from previous encounters it is clear that she defers to him. The
most effective way to present this treatment plan would be for the therapist to:
a. Present the patient with a written summary of the plan and ask her to look it over.
b. Ask the woman’s husband to step out of the room while the plan is being discussed
with the patient.
c. Explain the plan to the patient, maintaining eye contact with her at all times and
encouraging her to ask any questions she might have.
d. Explain the plan to the patient and her husband, and encourage both to ask any
questions they might have.
ANSWER: D
From the woman’s dress and previous behavior, it is likely that she is Muslim, with traditional
values and customs. Therefore, both she and her husband will find it inappropriate, if not
offensive, that he not be included in discussions of treatment. Cultural values must be taken
into account if truly effective patient treatment is to occur.
REF: pg. 5 OBJ: 4 | 7
TO GET ALL CHAPTERS EMAIL ME AT>>>>>
donc8246@gmail.com
26. The therapist enters the room of a 6-year-old victim of an automobile accident who is
unconscious and receiving ventilation therapy. The therapist assesses the patient, gives a
treatment, and suctions the patient. The patient’s mother then asks in a worried voice, “Is he
going to be all right?” The appropriate response for the therapist would be:
a. “I just looked at the CT scan of his brain, and I believe the swelling is going down.
He should recover within the next couple of weeks.”
b. “I’m unable to give you any information about your child’s condition.”
c. “I am just the respiratory therapist, and I really do not know anything.”
d. “I’m sorry, but our policy is that only the doctor can give you information about your
child’s prognosis. Let me step out and find out when the doctor will be back in the
unit.”
ANSWER: D
Response “a” is inappropriate because it is not within the scope of practice of an RT to render
medical opinions about a patient’s condition to the family. Answer “b,” while correct, is
unnecessarily abrupt. Answer “c” is both abrupt and is probably not true. Answer “d” meets
HIPAA and hospital policy requirements while also providing good patient care and customer
service.
REF: pgs. 4-5 OBJ: 6
27. In order to deliver effective patient education for use of a particular treatment, the first step
should be to:
a. Describe to the patient the equipment that will be used for the treatments.
b. Describe to the patient the medications that will be used for the treatments.
c. Assess the patient’s learning needs by identifying learning barriers, determining the
way the patient best learns, and evaluating the patient’s readiness to learn.
d. Describe to the patient the schedule for the treatments to be given.
ANSWER: C
Although the patient will eventually have to learn about equipment, medications, and
schedules, this learning will not occur effectively until the patient’s learning needs are
determined.
REF: pgs. 6-7 OBJ: 9
28. One effective teaching tool is the teach-back method. This is:
a. A technique where the teacher explains the procedure to the learner and has the
learner repeat the information in his or her own words.
b. A technique where the teacher explains the procedure to the learner and then has
the learner explain it to family members after the teacher has left the room.
c. A technique where the teacher gives the equipment to the learner and has the
learner experiment with it until he or she can use it correctly.
d. None of the above.
ANSWER: A
The teach-back method has the learner hear the explanation and then give a “return
demonstration” to the teacher to be sure that the learner has the correct information.
REF: pg. 7 OBJ: 9
TO GET ALL CHAPTERS EMAIL ME AT>>>>>
donc8246@gmail.com
29. Prior to discharge, patients should receive a written action plan that establishes treatment goals
and self-care activities. The acronym SMART is helpful in establishing the action plan. The “M”
in SMART stands for:
a. Meaningful (the goal pertains to the action plan).
b. Mastering (the goal).
c. Modular (the goal is divided into sections).
d. Measurable (the outcome should be measurable).
ANSWER: D
In order for a treatment to be effective, its outcome must be measurable.
REF: pg. 7 OBJ: 10
30. Failure of care providers to collaborate with one another potentially results in:
a. Patient safety placed at risk.
b. Duplication of effort by different caregivers.
c. Delivery of less-than-optimum care.
d. All of the above.
ANSWER: D
In addition to “a,” “b,” and “c,” lack of collaboration also may result in an increased length of
stay and wasted healthcare resources.
REF: pgs. 9-11 OBJ: 13 | 14 | 15
31. An RT enters a room for a patient’s second treatment of the day and notes that the patient is
dyspneic and tachycardic and has rales that can be heard throughout all lung fields but are
especially prominent in the bases. A treatment is given with little positive change. The
therapist should:
a. Note the treatment and its results in the patient chart.
b. Note the treatment and its results in the patient chart and tell the unit clerk to have
the nurse see the patient soon.
c. Find the patient’s nurse and together determine what is needed for the patient (e.g.,
different drug therapy, call physician, call rapid response team). Once the patient’s
condition has been adequately addressed, the therapist’s actions should be
documented in the chart.
d. Tell the patient you will check back with him in an hour to see how he is doing.
ANSWER: C
It is critical that results of an assessment and treatment be communicated to other members of
the health team, either in the chart or verbally if the situation demands immediate action.
Chart documentation should include findings of the assessment, results of any treatments
delivered, and actions taken to resolve the situation.
REF: pg. 10 OBJ: 15
32. Good communication is especially critical when “handing off” a patient to another caregiver at
the end of the shift. To ensure that adequate information is transmitted, the SBAR format is
useful. This acronym stands for situation, background, assessment, and .
a. Recommendation.
b. Results.
TO GET ALL CHAPTERS EMAIL ME AT>>>>>
donc8246@gmail.com
c. References.
d. Repeat (information)
ANSWER: A
The fourth letter in SBAR stands for recommendation.
REF: pg. 10 OBJ: 14
33. According to the American Association for Respiratory Care (AARC), patient discharge plans
should include which of the following?
a. Transportation for the patient when the patient goes home
b. Ascertaining that once the patient has been discharged, the patient has adequate
financial resources to pay rent and utilities
c. Notification of the patient’s family and/or other caregivers of the patient’s
imminent discharge
d. Methods for the ongoing assessment of outcomes
ANSWER: D
Discharge planning should focus on providing a continuum of care with transition from the
hospital to the alternate site.
REF: pg. 11 OBJ: 14
34. All of the following represent good hygiene practices except:
a. Gloves should be worn when touching a patient only when a caregiver is likely to
come into contact with secretions or infectious materials.
b. Hands should be washed when first encountering a patient, after leaving a patient,
and before and after any sterile or clean procedure is performed on a patient.
c. All infectious waste should be disposed of in proper waste containers.
d. Place all needles and blades in “sharps” containers when you are finished with
them.
ANSWER: A
Gloves should be worn whenever a patient is touched.
REF: pg. 9, Box 1-4 OBJ: 12
35. The main objective of the I Speak Up initiative from the National Institute of Health (NIH) is:
a. Be sure that all billing for patient medical expenses occurs correctly.
b. Let caregivers know that the family wants to be informed at all times.
c. Let the hospital know when ancillary services such as food quality and parking are
inadequate.
d. Help ensure that a patient’s care is as safe and effective as possible.
ANSWER: D
The I Speak Up initiative is a comprehensive program focused on making sure that patient care
is as safe and effective as possible. It especially emphasizes the importance of active patient and
family involvement in all aspects of patient care, including enhanced patient safety and reduction
in medical errors.
REF: pg. 8, Box 1-3 OBJ: 11
TO GET ALL CHAPTERS EMAIL ME AT>>>>>
donc8246@gmail.com
Chapter 2: The Medical History and the Interview
Test Bank
MULTIPLE CHOICE
1. Communication between two people can occur only if:
a. The speaker speaks clearly.
b. The receiver understands the message.
c. Each person is willing to listen to the other.
d. All of the above.
ANSWER: D
Communication occurs only if both parties speak clearly and listen carefully.
REF: pg. 16 OBJ: 1
2. Communication between individuals is affected by all of the following factors except:
a. The time of day.
b. The cultural heritage.
c. The religious beliefs.
d. The level of education.
ANSWER: A
The time of day is not likely to affect communication in a significant way.
REF: pg. 16 OBJ: 2
3. When one is conducting an interview with a patient, which of the following points is most
important in facilitating an effective interaction with the patient?
a. Recognizing the nonverbal signals that the patient is sending
b. Your ability to project a sense of undivided interest in the patient
c. Introducing yourself appropriately at the beginning of the interview
d. Answering all of the patient’s questions completely without using jargon
ANSWER: B
Patients can sense when we are distracted and will not communicate well in such a situation.
Glancing out the window or at the television will tell the patient that you are distracted and are not
that interested in what he or she has to say.
REF: pgs. 16-17 OBJ: 3
4. Which of the following types of questions are preferred for all interactions with a patient?
a. Direct questions
b. Neutral questions
c. Indirect questions
d. Open-ended questions
ANSWER: B
Neutral questions encourage the patient to respond with sentences and honest answers.
REF: pg. 17 OBJ: 3
TO GET ALL CHAPTERS EMAIL ME AT>>>>>
donc8246@gmail.com
5. If a patient is unable to provide an accurate history, the respiratory therapist (RT) should:
a. Ask a family member or friend to supply the information.
b. Ask shorter and more direct questions to determine the cause of the problem.
c. Proceed to treat the patient’s symptoms based on information already obtained.
d. Refuse to treat the patient because effective therapy cannot be initiated without a
complete history.
ANSWER: A
In many situations, the patient is unable to answer questions. Family members are often the next
best source for important information.
REF: pg. 18 OBJ: 4
6. Which of the following should the RT keep in mind when obtaining a pulmonary history?
a. Assessment usually is limited to the respiratory system.
b. Evaluation of the patient’s entire health status is essential.
c. Signs and symptoms of pulmonary disease will rarely be seen outside the
cardiopulmonary system.
d. Patients with long-standing chronic disease can give a detailed account of how their
lives have changed and of the signs and symptoms that the disease has caused.
ANSWER: B
Pulmonary problems often cause health issues in other body systems, and health problems in
other systems often cause pulmonary problems.
REF: pg. 18 OBJ: 2 | 6
7. Obtaining background information during an interview is very important because it allows the
interviewer to:
a. Learn the impact of culture, relationships, and finances on his or her health.
b. Predict whether the patient is willing to cooperate in the treatment of his or her
disease.
c. Develop a basic understanding of the patient’s experience with his or her disease.
d. Achieve all of the above.
ANSWER: D
Background information is often overlooked but is important for assessing the impact of
chronic illness on the patient and for identifying how the patient is coping with the illness. It also
tells the interviewer whether the patient is able to cooperate with the treatment plan.
REF: pg. 18 OBJ: 2 | 6
8. Screening information is:
a. Obtained at health fairs to determine whether the person should see a physician.
b. Designed to uncover problem areas that the patient forgot to mention or omitted.
c. Given to the interviewer by the patient at the very beginning of the interview
process.
d. Obtained by the triage nurse in the emergency department to determine how life-
threatening the patient’s symptoms are.
ANSWER: B
TO GET ALL CHAPTERS EMAIL ME AT>>>>>
donc8246@gmail.com
Screening information is designed to identify important facts that the patient may have
overlooked in the interview.
REF: pgs. 18-19 OBJ: 2
9. The review of systems is very important because it provides the interviewer with:
a. Information grouped by major organs and physiologic systems.
b. Additional subjective information about the patient’s problem.
c. A combination of subjective and objective data in a narrative form.
d. Information relevant to the patient’s problem that may have been overlooked.
ANSWER: D
The review of systems helps identify key information that may have been overlooked in the
interview.
REF: pg. 19 OBJ: 2
10. A pertinent negative is defined as:
a. Any negative response made by the patient during the interview.
b. Refusal by the patient to answer questions about a certain topic.
c. Any negative response by the patient to an important question about possible
symptoms.
d. A negative response by a patient to a therapist who asks whether the patient needs a
breathing treatment.
ANSWER: C
If the patient appears to have pneumonia but denies having a cough, the patient’s negative
response to the question about coughing would be a pertinent negative.
REF: pg. 19 OBJ: 3
11. A pertinent positive is defined as:
a. Any positive response made by the patient during the interview.
b. A direct question asked during the interview that elicits a positive response.
c. An affirmative response by the patient when asked whether he or she needs a
breathing treatment.
d. An affirmative response to an important interview question about the patient’s
symptoms.
ANSWER: D
A positive response from the patient regarding a symptom associated with the possible
diagnosis would be a pertinent positive.
REF: pg. 19 OBJ: 3
12. The main purpose of the chief complaint is to:
a. Give a brief explanation about why the patient sought health care.
b. Direct the interviewer to the organ system in which the problem is located.
c. Present a diagnosis that is based on information obtained during the interview.
d. List symptoms in order from most severe to least severe according to organ system
involvement.
TO GET ALL CHAPTERS EMAIL ME AT>>>>>
donc8246@gmail.com
ANSWER: A
The chief complaint explains why the patient sought medical help.
REF: pg. 19 OBJ: 2 | 3
13. Which of the following cardiopulmonary conditions would not be found in the chief
complaint list?
a. Asthma
b. Wheezing
c. Chest pain
d. Hemoptysis
ANSWER: A
Asthma is a diagnosis and not a complaint.
REF: pg. 19 | pg. 21 OBJ: 2
14. When a patient is interviewed so the chief complaint can be determined, the best questions
that can be asked to elicit this information are:
a. Direct questions.
b. Neutral questions.
c. Open-ended questions.
d. A combination of the above.
ANSWER: D
Use of a variety of types of questions usually yields the best interview results.
REF: pg. 17 | pg. 19 OBJ: 2
15. Which of the following would not be found in the past medical history?
a. Injuries and accidents
b. Surgeries and hospitalizations
c. Associated symptoms and aggravating factors
d. Over-the-counter medications, vitamins, and “home remedies”
ANSWER: C
Associated symptoms and aggravating factors would be found in the history of present illness.
REF: pg. 22 OBJ: 2
16. Which of the following formulas should be used to calculate the pack-year history of cigarette
consumption?
a. Packs per day times Years smoked
b. Packs per day minus Years smoked
c. Packs per day plus Years smoked
d. Packs per day plus Years smoked
ANSWER: A
Pack-years is the standard way to document a patient’s smoking history. This is determined by
multiplying the number of packs smoked times the number of years smoked.
REF: pg. 24 OBJ: 2
TO GET ALL CHAPTERS EMAIL ME AT>>>>>
donc8246@gmail.com
17. One disadvantage of using the pack-year method for calculating cigarette consumption is that:
a. The method is not used widely throughout the United States.
b. Advanced mathematical calculations are required to obtain the figure.
c. The method does not reveal how many packs per day were smoked over how many
years.
d. The values are more meaningful to RTs and physicians than to other healthcare
providers.
ANSWER: C
Although pack-years is an industry standard for documenting quantity of tobacco
consumption, this value does not provide the details.
REF: pg. 24 OBJ: 2
18. Which of the following is not a reason for obtaining a family history?
a. To assess the current health status of the extended family
b. To learn about the health status of the patient’s blood relatives
c. To determine whether the patient is adopted
d. To identify the presence in the family of diseases with a hereditary tendency
ANSWER: C
The reason for obtaining a family history is to identify blood relatives who may have a disease
that the patient has inherited.
REF: pg. 25 OBJ: 2
19. Which of the following diseases would not be recorded in the family history as a hereditary
disorder?
a. Asthma
b. Pneumonia
c. Cystic fibrosis
d. Alpha1-antitrypsin deficiency
ANSWER: B
Pneumonia is not hereditary.
REF: pg. 25 OBJ: 2
20. Modern-day office workers may be exposed to which of the following occupational and
environmental diseases?
a. Bagassosis
b. Monday fever
c. Pneumoconiosis
d. Sick building syndrome
ANSWER: D
Sick building syndrome occurs when employees are exposed to low doses of toxic gases in a
building that is airtight and has poor ventilation.
REF: Table 2-1, pg. 27 OBJ: 2
TO GET ALL CHAPTERS EMAIL ME AT>>>>>
donc8246@gmail.com
21. Which of the following symptoms is often seen in patients with tight building syndrome?
a. Headache
b. Stuffy nose
c. Cough
d. All of the above
ANSWER: D
Tight building syndrome can cause a variety of symptoms.
REF: Table 2-1, pg. 27 OBJ: 2
22. What pulmonary disorder is associated with visiting or living in Ohio, Maryland, and the
central Mississippi Valley?
a. Blastomycosis
b. Histoplasmosis
c. Coccidioidomycosis
d. Silicosis
ANSWER: B
Histoplasmosis occurs only in the midwestern regions of the country.
REF: Table 2-1, pg. 28 OBJ: 2
23. Who writes the initial admission note?
a. The physician
b. The head nurse
c. The head RT
d. Any of the above
ANSWER: A
Only the physician writes the initial admission note, which describes why the patient is being
admitted.
REF: pg. 28 OBJ: 2 | 6
24. Who writes the progress notes each day?
a. The physician
b. The physical therapist
c. The nurse
d. Any of the above
ANSWER: D
Any clinician who provides treatment to the patient can write a progress note in the chart.
REF: pg. 28 OBJ: 2 | 6
25. A good interview should contain all of the following elements except:
a. The interviewer should dress and act professionally.
b. The interviewer should project a sense of undivided interest.
c. The interviewer should use a formal speaking style.
d. The interviewer should respect the patient’s beliefs and attitudes.
ANSWER: C
TO GET ALL CHAPTERS EMAIL ME AT>>>>>
donc8246@gmail.com
The interviewer should use an informal, relaxed, conversational style.
REF: pg. 17 OBJ: 2 | 3
26. In the physical examination, objective data gathered are referred to as:
a. Measurements.
b. Symptoms.
c. Variables.
d. Signs.
ANSWER: D
Objective data are referred to as “signs” in the physical examination. These are data that can be
perceived by the examiner, either by measurements or observations.
REF: pg. 19 OBJ: 5
27. In the physical examination, subjective data gathered are referred to as:
a. Measurements.
b. Symptoms.
c. Variables.
d. Signs.
ANSWER: B
Subjective data in the physical examination are referred to as symptoms. These are data that
can only be perceived by the patient. A good interviewer, however, is able to ask questions
that prompt the patient to give complete and accurate descriptions of his or her symptoms.
REF: pg. 19 OBJ: 5
28. An RT examining a patient auscultates wheezes in the right lower lobe (RLL). This would be an
example of:
a. Objective data (a sign).
b. Objective data (a symptom).
c. Subjective data (a sign).
d. Subjective data (a symptom).
ANSWER: A
Because wheezes are perceived by the examiner, this would be an example of a sign.
REF: pg. 19 OBJ: 5
29. A patient being interviewed says that he wakes up nearly every night feeling very hot and with his
body bathed in sweat. This fact, along with his hemoptysis, is a very strong indication that he
may have tuberculosis. The night sweats would be an example of:
a. Objective data (a sign).
b. Objective data (a symptom).
c. Subjective data (a sign).
d. Subjective data (a symptom).
ANSWER: D
Because the night sweats are perceived only by the patient and then described to the examiner, they
would be an example of subjective data (a symptom).
TO GET ALL CHAPTERS EMAIL ME AT>>>>>
donc8246@gmail.com
REF: pg. 19 OBJ: 5
30. A “constitutional” symptom would include all of the following except:
a. Chills and fever.
b. Anorexia and/or weight loss.
c. Fatigue.
d. Wheezing.
ANSWER: D
Constitutional symptoms generally involve the whole body and are not specific to individual
systems. Wheezing is definitely specific to the respiratory system.
REF: pg. 21 OBJ: 6
31. All of the following questions would be effective in eliciting additional information about a
patient’s location and level of pain except for question .
a. Earlier, you briefly mentioned some pain around your lower ribs. Could you tell me
more about it?
b. Earlier, you briefly mentioned some pain around your lower ribs. Could you point
to exactly where it is?
c. Earlier, you briefly mentioned some pain around your lower ribs. What do you
think is causing it?
d. Earlier, you briefly mentioned some pain around your lower ribs. Could you rate
this pain for me on a scale of 1 to 10, with 1 meaning no pain and 10 meaning the
worst pain possible?
ANSWER: C
A good interviewer is looking for a precise description of a symptom. It is unlikely that the
patient has any idea what is causing the pain.
REF: pg. 21 OBJ: 2
32. A history of a patient’s tobacco use is important for all of the following reasons except:
a. There is a strong relationship between smoking and chronic obstructive pulmonary
disease (COPD).
b. There is a strong relationship between smoking and the use of illicit drugs such as
marijuana, cocaine, and heroin.
c. There is a strong relationship between smoking and cardiovascular disease.
d. There is a strong relationship between smoking and lung cancer.
ANSWER: B
There is no published evidence that shows that smoking increases the likelihood that a person
will use illicit drugs. There is ample evidence of the relationship between smoking and COPD,
cardiovascular disease, and lung cancer.
REF: pg. 22 OBJ: 2 | 3 | 6
33. Monday fever is:
a. The term describing workers who habitually do not want to return to work after the
weekend.
b. The tendency of workers to have poor levels of concentration in the workplace after
TO GET ALL CHAPTERS EMAIL ME AT>>>>>
donc8246@gmail.com
the weekend.
c. An example of a hypersensitivity reaction to a toxic inhaled substance that is worst
after the initial exposure but becomes progressively less pronounced as the exposure
continues.
d. None of the above.
ANSWER: C
First exposure to a toxic inhaled substance may cause an immediate reaction (as on Monday
following a weekend), which gradually becomes less severe with continued exposure (i.e.,
exposure throughout the rest of the week).
REF: pg. 27 OBJ: 6
34. In which section of the patient record should the following entry appear? “Day 4 of
hospitalization. Patient febrile (39o
C). Rales in both lung bases; sputum moderate, thick, green,
tinged with blood. Plan: Continue ordered antibiotic therapy. Bronchoscopy tomorrow in AM.”
a. Admission note
b. Physician orders
c. Progress notes
d. Discharge plan
ANSWER: C
The physician should see the hospitalized patient at least once daily to identify the patient’s
general condition, progress, and response to treatment. These findings are summarized in the
progress notes.
REF: pg. 28 OBJ: 6 | 7
35. The RT has been called STAT to the bedside of a patient who is in extreme respiratory distress in
the intensive care unit (ICU). Shortly after arriving, the patient’s cardiac monitor shows a pattern
of ventricular fibrillation, and shortly thereafter the patient ceases breathing. The nurse states
that the patient has a do not resuscitate (DNR) order. The RT should take which of the
following actions?
a. Do nothing, because a DNR order means that no resuscitative measures should be
attempted.
b. Call a full code and institute full resuscitative measures because the therapist knows
that the physician who wrote the DNR order is not working in the unit this week.
c. Call a code so that cardiac medications can be given to correct the ventricular
fibrillation; the DNR order means that the patient cannot be intubated.
d. Place a non–rebreather mask on the patient for “comfort measures.”
ANSWER: A
A DNR order signed by a physician means that no resuscitative measures should be undertaken.
REF: pg. 28 OBJ: 8
TO GET ALL CHAPTERS EMAIL ME AT>>>>>
donc8246@gmail.com
Chapter3:CardiopulmonarySymptoms Test
Bank
MULTIPLE CHOICE
1. Which of the following sequences of events best describes the cough mechanism?
a. Inspiration, closure of glottis, forceful opening of glottis
b. Opening of glottis, relaxation of diaphragm, closure of glottis
c. Inspiration, forceful opening of glottis, contraction of diaphragm
d. Opening of glottis, contraction of diaphragm, explosive release of trapped
intrathoracic air
ANSWER: A
The cough mechanism can be divided into three phases:
1. Inspiratory phase: Reflex opening of the glottis and contraction of the diaphragm and the
thoracic and abdominal muscles cause a deep inspiration with a concomitant increase in lung
volume accompanied by an increase in the caliber and length of the bronchi.
2. Compression phase: Closure of the glottis and relaxation of the diaphragm while the
expiratory muscles contract against the closed glottis can generate very high intrathoracic
pressures and narrowing of the trachea and bronchi.
3. Expiratory phase: Opening of the glottis and explosive release of trapped intrathoracic air
occur, along with vibration of the vocal cords and of the mucosal lining of the posterior
laryngeal wall, which shakes secretions loose from the larynx and moves undesired material out
of the respiratory tract.
REF: pg. 33 OBJ: 1
2. Which of the following mechanisms does not explain why patients with chronic obstructive
pulmonary disease (COPD) have a poor cough?
a. Weak muscles of breathing
b. Increased elastic recoil
c. Airway obstruction
d. Glottis function failure
ANSWER: B
The effectiveness of a cough is reduced when one or more of the following conditions exist:
(1) weakness of the inspiratory or expiratory muscles;
(2) inability of the glottis to open or close correctly;
(3) obstruction, collapsibility, or alteration in shape or contours of the airways;
(4) decrease in lung recoil, as occurs with emphysema; and
(5) abnormal quantity or quality of mucus production (e.g., thick sputum).
REF: pg. 33 OBJ: 1
3. Chronic productive cough is caused most commonly by which of the following clinical
conditions?
a. Postnasal drip
b. Viral infection
c. Upper airway infection
d. Bronchopulmonary disease
TO GET ALL CHAPTERS EMAIL ME AT>>>>>
donc8246@gmail.com
ANSWER: A
Chronic persistent cough is caused most commonly by postnasal drip syndrome, followed by
acute asthma, acute exacerbation of COPD, allergic rhinitis, gastroesophageal reflux disease
(GERD), chronic bronchitis, bronchiectasis, and other conditions such as left heart failure,
bronchogenic cancer, and sarcoidosis.
REF: pg. 34 OBJ: 1
4. Which of the following is not a complication of forceful coughing?
a. Rib fracture
b. Pneumothorax
c. Pleural effusion
d. Torn chest muscle
ANSWER: C
The vigorous muscular activity and high intrathoracic pressures created by forceful coughing
may produce a number of complications, such as torn chest muscles, rib fractures, disruption of
surgical wounds, pneumothorax or pneumomediastinum, syncope (fainting), arrhythmia,
esophageal rupture, and urinary incontinence.
REF: pg. 34 OBJ: 1
5. Which of the following conditions is not associated with a characteristic “hacking” cough?
a. Smoking
b. Viral infection
c. Nervous habit
d. Lung cancer
ANSWER: D
Hacking (frequent brief periods of coughing or clearing the throat) may be dry and may be the
result of smoking, a viral infection, a nervous habit, or difficult-to-move secretions, as occur
with postnasal drip.
REF: pg. 34 OBJ: 1
6. Which of the following is the best definition of sputum?
a. Secretions
b. Thick tenacious secretions
c. Secretions from the lungs and lower airways
d. Secretions from the nose, mouth, and tracheobronchial tree
ANSWER: D
Sputum is the substance expelled from the tracheobronchial tree, pharynx, mouth, sinuses, and
nose by coughing or clearing the throat.
REF: pg. 35 OBJ: 1
7. What term is used to describe secretions strictly from the lungs and lower airways?
a. Phlegm
b. Sputum
c. Mucus
TO GET ALL CHAPTERS EMAIL ME AT>>>>>
donc8246@gmail.com
d. Empyema
ANSWER: A
The term phlegm refers strictly to secretions from the lungs and tracheobronchial tree.
REF: pg. 35 OBJ: 1
8. Excessive sputum production is associated with all the following conditions except:
a. Allergies.
b. Cigarette smoking.
c. Airway infection.
d. Pleural infection.
ANSWER: D
Excessive sputum production is caused most often by inflammation of the mucous glands that
line the tracheobronchial tree. Inflammation of these glands occurs most often with airway
infection, cigarette smoking, and allergies.
REF: pg. 35 OBJ: 1
9. Which of the following terms best describes foul-smelling sputum?
a. Fetid
b. Mucoid
c. Purulent
d. Mucopurulent
ANSWER: A
Copious, foul-smelling (fetid) sputum that separates into layers when standing occurs with
bronchiectasis and lung abscess when the patient’s position is changed.
REF: pg. 36 OBJ: 1
10. A patient presents in the emergency department with blood-tinged sputum. The term
associated with such sputum is:
a. Gelatinous.
b. Hemoptysis.
c. Hematemesis.
d. Mucopurulent.
ANSWER: B
Hemoptysis, expectoration of sputum that contains blood, varies in severity from slight
streaking to frank bleeding.
REF: pg. 36 OBJ: 1
11. Which of the following conditions is believed to be the most common cause of hemoptysis?
a. Tuberculosis
b. Erosive bronchitis
c. Bronchogenic carcinoma
d. Pneumonia
ANSWER: B
TO GET ALL CHAPTERS EMAIL ME AT>>>>>
donc8246@gmail.com
Erosive bronchitis in smokers with chronic bronchitis now accounts for nearly half the cases of
hemoptysis.
REF: pg. 36 OBJ: 1
12. Which of the following definitions is consistent with massive hemoptysis?
a. 400 mL in 3 hours
b. 400 mL in 12 hours
c. 600 mL in 48 hours
d. 600 mL in 72 hours
ANSWER: A
Massive hemoptysis (400 mL in 3 hours, or more than 600 mL in 24 hours) is seen with lung
cancers, tuberculosis, bronchiectasis, and trauma.
REF: pg. 37 OBJ: 1
13. Hemoptysis in a patient with sudden onset of chest pain who is at risk for venostasis is
suggestive of which of the following conditions?
a. Pneumonia
b. Pulmonary embolism
c. Myocardial infection
d. Atelectasis
ANSWER: B
Hemoptysis associated with sudden onset of chest pain and dyspnea in a patient at risk for
venous stasis of the legs must lead to assessment of that patient for pulmonary embolism and
possible infarction.
REF: pg. 37 OBJ: 1
14. In the presence of nausea and vomiting, a history of cirrhosis of the liver suggests which of
the following organs as the source of hematemesis?
a. Kidneys
b. Duodenum
c. Esophagus
d. Liver
ANSWER: C
The presence of symptoms such as nausea and vomiting, especially with a history of alcoholism
or cirrhosis of the liver, may suggest the esophagus or stomach as the source of hematemesis.
REF: pg. 37 OBJ: 1
15. Dyspnea is defined as:
a. A visible increase in the work of breathing.
b. An increase in respiratory rate and depth.
c. Breathlessness as perceived by the patient.
d. Difficulty breathing in the upright position.
ANSWER: C
TO GET ALL CHAPTERS EMAIL ME AT>>>>>
donc8246@gmail.com
Dyspnea (dys, difficult; pnea, breathing) is defined as a subjective experience of breathing
discomfort that consists of qualitatively distinct sensations that vary in intensity.
REF: pg. 38 OBJ: 1
16. Which of the following grading systems is useful in qualifying the degree of dyspnea?
a. Murray lung injury score
b. Modified Borg scale
c. Dubowitz score
d. Asthma score
ANSWER: B
The modified Borg scale, as shown as Table 3-5, pg. 39, uses a 0 to 10 grading system, with
descriptive terms to depict the perceived intensity of a symptom such as dyspnea after a
specified task.
REF: Table 3-5, pg. 39 OBJ: 1
17. Dyspnea tends to occur when which of the following is present?
a. Work of breathing that is adequate for a given level of exertion
b. Decrease in ventilatory drive to breathe
c. Increase in ventilatory capacity
d. Increase in ventilatory drive to breathe
ANSWER: D
It is helpful to remember that patients with respiratory disorders will complain of dyspnea when
any of the following is present alone or in combination:
1. The work of breathing is abnormally high for the given level of exertion. This is common
with narrowed airways, as in asthma, and when the lung is stiff, as in pneumonia.
2. The ventilatory capacity is reduced. This is common when the vital capacity is abnormally
low, as is seen in patients with neuromuscular disease.
3. The drive to breathe is elevated beyond normal (e.g., hypoxemia, acidosis, exercise).
REF: pg. 39 OBJ: 1
18. Breathing at a rate and depth in excess of the body’s metabolic need is known as:
a. Hyperventilating.
b. Hyperpnea.
c. Platypnea.
d. Psychogenic dyspnea.
ANSWER: A
Hyperventilating is breathing at a rate and depth in excess of the body’s metabolic need, which
causes a decrease in arterial carbon dioxide (PaCO2) and results in a decrease in cerebral blood
flow.
REF: pg. 40 OBJ: 1
19. All of the following conditions are associated with acute dyspnea in children except:
a. Asthma.
b. Cystic fibrosis.
TO GET ALL CHAPTERS EMAIL ME AT>>>>>
donc8246@gmail.com
c. Bronchiolitis.
TO GET ALL CHAPTERS EMAIL ME AT>>>>>
donc8246@gmail.com
d. Croup.
ANSWER: B
Acute dyspnea in children is associated most frequently with asthma, bronchiolitis, croup, and
epiglottitis.
REF: pg. 41 OBJ: 1
20. Which of the following diseases is one of the most common causes of chronic dyspnea in
adults?
a. Asthma
b. Bronchiolitis
c. Cystic fibrosis
d. Congestive heart failure (CHF)
ANSWER: D
COPD and chronic CHF are the most common causes of chronic dyspnea in adults.
REF: pg. 41 OBJ: 1
21. Paroxysmal nocturnal dyspnea is associated commonly with which of the following
conditions?
a. Asthma
b. COPD
c. CHF
d. Obstructive sleep apnea
ANSWER: C
Paroxysmal nocturnal dyspnea (PND) is the sudden onset of difficulty in breathing that occurs
when a sleeping patient is in the recumbent position. It often is associated with coughing and is
relieved when the patient assumes an upright position. In patients with CHF, PND usually
occurs 1 to 2 hours after lying down and is caused by the gradual transfer to the lungs of fluid in
the lower extremities.
REF: pg. 41 OBJ: 1
22. An inability to breathe while lying down is known as:
a. Dyspnea.
b. Platypnea.
c. Orthopnea.
d. PND.
ANSWER: C
Orthopnea is the inability to breathe when lying down.
REF: pg. 41 OBJ: 1
23. Chest pain is the cardinal symptom of which of the following diseases?
a. Heart disease
b. Pleural disease
c. Chest wall disease
d. Neuromuscular disease
TO GET ALL CHAPTERS EMAIL ME AT>>>>>
donc8246@gmail.com
ANSWER: A
Chest pain is the cardinal symptom of heart disease.
REF: pg. 42 | pg. 44 OBJ: 1
24. What is the difference between pleuritic and nonpleuritic chest pain?
a. Pleuritic pain is inspiratory; nonpleuritic pain is expiratory.
b. Pleuritic pain is sharp and stabbing; nonpleuritic pain is dull and crushing.
c. Pleuritic pain usually involves the pleura; nonpleuritic pain involves the lungs.
d. Pleuritic pain involves the lung itself; nonpleuritic pain involves the chest wall.
ANSWER: B
Pleuritic pain, often described as inspiratory pain, is the most common symptom of disease; it is
the result of inflammation of the pleura (pleurisy). It is sharp, often abrupt in onset, and severe
enough to cause the patient to seek medical help (often within hours of onset). It increases with
inspiration, a cough, a sneeze, a hiccup, or laughing. Pleuritic pain usually is localized to one side
of the chest—frequently, the lower lateral aspect. It may be only partially relieved by splinting
and pain medication. Pleuritic pain increases with pressure and movement but not to the same
degree as pain originating from the outer chest wall. In contrast, the lung parenchyma and the
visceral pleura that cover the lungs are relatively insensitive to pain; therefore, pain with
breathing usually indicates involvement of the parietal pleura.
REF: pg. 44 OBJ: 1
25. All the following pulmonary conditions are associated with syncope except:
a. Pulmonary embolism.
b. Prolonged bouts of coughing.
c. Hyperoxia.
d. The Valsalva maneuver.
ANSWER: C
Pulmonary causes of syncope include pulmonary embolism (obstruction of blood flow from the
right heart to the left heart), prolonged bouts of coughing (tussive syncope), and hypoxia (low
levels of blood oxygen) or hypocapnia (low levels of carbon dioxide). Holding one’s breath
following a deep inspiration (Valsalva maneuver) results in high intrathoracic pressure and
decreased venous return to the heart.
REF: pg. 47 OBJ: 1
26. Cough syncope is associated most often with which of the following types of patients?
a. Middle-aged men with underlying COPD
b. Pregnant women
c. Children with asthma
d. Older obese men with diabetes
ANSWER: A
Cough (tussive) syncope is the transient loss of consciousness that may follow severe
coughing. It occurs most commonly in middle-aged men with underlying COPD who are
outgoing and moderately obese and have a large appetite for food, alcohol, and smoking.
TO GET ALL CHAPTERS EMAIL ME AT>>>>>
donc8246@gmail.com
REF: pg. 47 OBJ: 1
27. The presence of anasarca is associated commonly with which of the following conditions?
a. Dyspnea
b. Edema
c. Hepatomegaly
d. Pleural effusion
ANSWER: B
Edema is soft tissue swelling that results from an abnormal accumulation of fluid. It may be
generalized (anasarca), may appear only in dependent body areas (feet and ankles in ambulatory
patients; sacral area in patients restricted to bed rest), or may be limited to a single extremity or
organ (such as pulmonary edema).
REF: pg. 48 OBJ: 1
28. Which of the following conditions is often associated with right heart failure?
a. Organomegaly
b. Hepatomegaly
c. Loud A2 heart sound
d. Shallow breathing
ANSWER: B
As right heart failure worsens, dependent edema is no longer relieved by rest or changing
position, and edema occurs in the abdominal organs, as well as the extremities and dependent
areas of the body. As the liver becomes enlarged (hepatomegaly) as a result of edema, the
patient may complain of pain just below the ribs on the right side (right upper quadrant pain).
REF: pg. 48 OBJ: 1
29. What is the most common manifestation of infection in a patient with pulmonary disease?
a. Fever
b. Cough
c. Diaphoresis
d. Increased sputum production
ANSWER: A
Fever is the most common manifestation of infection. It usually is assumed to be caused by an
infectious process until proven otherwise.
REF: pg. 49 OBJ: 1
30. The presence of early morning headache may be caused by which of the following conditions?
a. Hypoglycemia
b. Anoxia
c. Hypoxemia
d. Hypercapnia
ANSWER: D
TO GET ALL CHAPTERS EMAIL ME AT>>>>>
donc8246@gmail.com
Because hypercapnia worsens during sleep in patients with pulmonary disorders, early morning
headaches may be the first indication that the patient is retaining abnormally high amounts of
carbon dioxide. If hypercapnia persists, headaches may be present throughout the day.
REF: pg. 51 OBJ: 1
31. The peak incidence of snoring in adult males occurs at ages years.
a. 60 to 64
b. 50 to 59
c. 51 to 55
d. 45 to 50
ANSWER: B
The peak incidence of snoring occurs at ages 50 to 59 years in males and at ages 60 to 64
years in women.
REF: pg. 52 OBJ: 1
32. Gastroesophageal reflux disease (GERD) is defined as reflux that occurs more than:
a. Twice a month.
b. Once a month.
c. Twice a week.
d. Once a week.
ANSWER: C
Persistent reflux that occurs more than twice a week is often referred to as GERD.
REF: pg. 53 OBJ: 1
33. You are a respiratory therapist working in a pediatrician’s office. You see a 16-year-old girl
who presents with two chief complaints but otherwise appears to be healthy. The complaints
are (1) extremely heavy menstrual bleeding and (2) severe dyspnea following relatively mild
exercise. The dyspnea has been getting progressively worse over the past 3 months. Which of
the following laboratory studies would you suggest to the physician in order to determine the
cause of the dyspnea?
a. Pulmonary function studies
b. Hemoglobin and hematocrit studies
c. Sputum culture and sensitivity studies
d. Chest x-ray
ANSWER: B
Dyspnea during exercise in an otherwise healthy individual may be caused by extreme anemia
(hemoglobin <7g/dL). In this case, anemia would likely be associated with the extremely heavy
menstrual bleeding.
REF: pg. 42 OBJ: 1
34. A sharp, “viselike” chest pain radiating usually down the left arm and possibly spreading to
the shoulders, neck, and jaw is called:
a. Pleuritic pain.
b. Thoracic pain.
TO GET ALL CHAPTERS EMAIL ME AT>>>>>
donc8246@gmail.com
c. Angina.
d. Epigastric pain.
ANSWER: C
Angina is the pain described and is an indication of myocardial ischemia. It often precedes a
myocardial infarction.
REF: pg. 44 OBJ: 1
35. A patient comes to the emergency department following an episode of unexpected fainting
(syncope). In order to determine the potential cause, all of the following could contribute
valuable information except:
a. A sensation of an irregular heartbeat or history of cardiac arrhythmias.
b. The medications taken.
c. The activities at the time of the episode.
d. The time of day.
ANSWER: D
It is unlikely that the time of day will have any bearing on the cause of a syncopal episode,
especially when factors such as arrhythmias present, medications being taken, and activities
being undertaken at the time of the episode are taken into account.
REF: pg. 47 OBJ: 1
36. A patient presents at the physician’s office with a complaint of severe, aching pain in her right
leg. An examination of the patient’s lower body reveals that the right ankle is badly bruised and
severely swollen. The patient states that she fell 2 days ago and “sprained” her ankle but did not
seek medical attention at that time. She has been mostly either sitting or lying down since the
accident. Her leg pain and swelling could be due to:
a. Her sprained ankle.
b. Deep vein thrombosis (DVT) in her right leg.
c. A side effect from the bruising.
d. All of the above.
ANSWER: D
Any soft tissue, bone, or joint injury in the leg will lead to swelling in the area surrounding the
injury. In addition, venous stasis caused by the inactivity as well as bruising can lead to formation
of DVT. This is especially dangerous because a deep vein thrombus can break loose and cause a
pulmonary embolus, a potentially fatal condition.
REF: pg. 48 OBJ: 1
37. A patient is being evaluated for surgery. The therapist hears rales in the lung bases and notes
prominent swelling of the ankles. She presses a finger into the edema and notices that a
depression remains when she lifts her finger away. The depression is clearly visible for slightly
more than 1 minute. This would be classified as pitting edema:
a. 1+.
b. 2+.
c. 3+.
d. 4+.
ANSWER: C
TO GET ALL CHAPTERS EMAIL ME AT>>>>>
donc8246@gmail.com
Pitting edema that remains visible for 1 to 2 minutes is classified as 3+ pitting edema.
REF: Table 3-12, pg. 49 OBJ: 1
38. A patient who appears emaciated presents to the emergency department with a history of low-
grade, intermittent, afternoon fevers; frequent, paroxysmal bouts of coughing; hemoptysis; and
night sweats. The following should be considered as part of the differential diagnosis (a list of
possible disease states that could account for the observed signs and symptoms):
a. Tuberculosis
b. Pneumonia
c. Some types of cancers (e.g., lymphoma)
d. All of the above
ANSWER: D
Choices a, b, and c could result in the observed symptoms. However, the most likely choice is
tuberculosis.
REF: pg. 36 OBJ: 1
39. Snoring is caused by all of the following except:
a. Excessive daytime sleepiness.
b. Obesity.
c. Excessive alcohol consumption.
d. Use of sleeping medications.
ANSWER: A
Excessive daytime sleepiness is a result of obstructive sleep apnea and is associated with snoring;
however, it does not cause snoring. Snoring is due primarily to narrowing of the hypopharynx.
This can result from extra tissue in the neck that accompanies obesity, as well as excessive
relaxation of the muscles surrounding the hypopharynx caused by alcohol intoxication or
sleeping medications.
REF: pg. 52 OBJ: 1
40. Gastroesophageal reflux is caused by:
a. High activity levels in gastric muscles.
b. Blockage of the duodenum.
c. Abnormal opening of the pyloric sphincter.
d. Abnormal opening of the cardiac (lower esophageal) sphincter.
ANSWER: D
Inappropriate opening of the cardiac sphincter allows stomach contents to move into the
esophagus, resulting in heartburn. Gastric contents reaching the larynx may be aspirated and
cause symptoms of gastroesophageal reflux disease (GERD).
REF: pg. 53 OBJ: 1
TO GET ALL CHAPTERS EMAIL ME AT>>>>>
donc8246@gmail.com
Chapter4: Vital Signs
Test Bank
MULTIPLE CHOICE
1. Which of the following is not classified as a classic vital sign?
a. Pulse
b. Sensorium
c. Blood pressure
d. Respiratory rate
ANSWER: B
The four classic vital sign measurements are temperature, pulse, respirations, and blood
pressure.
REF: pg. 57 OBJ: 1
2. How often should vital signs be recorded in a patient who is hospitalized but is not in
intensive care?
a. Every hour
b. Every 2 hours
c. Every 4 to 6 hours
d. Every 12 hours
ANSWER: C
For patients who are hospitalized but are not in intensive care, routine vital signs are recorded
most commonly every 4 to 6 hours (at the beginning of a shift and in the middle of a shift).
REF: pg. 58 OBJ: 1
3. Which of the following statements regarding vital signs is most accurate?
a. Vital signs are used only to establish a baseline.
b. Vital signs monitor only the response to therapy.
c. Trends in vital signs are far more important than a single measurement.
d. Vital signs at baseline should never be compared with those that were obtained 24
hours earlier.
ANSWER: C
The initial reading generally is referred to as the baseline measurement. A series of vital sign
measurements over time establishes a trend and is far more important clinically than any single
measurement. Each time vital signs are measured, they should be compared with baseline values
and the most recent measurements. Sometimes the patient’s condition may be changing slowly,
and comparison with one or two previous measurements does not indicate a trend, whereas
comparison over the span of an entire shift or 24-hour period may indicate clearly that the
patient is deteriorating slowly.
REF: pg. 58 OBJ: 2
4. Which of the following statements best reflects the importance of comparing changes in vital
signs?
REF: pg. 59 OBJ: 2
TO GET ALL CHAPTERS EMAIL ME AT>>>>>
donc8246@gmail.com
a. It allows assessment of important subjective patient data.
b. It allows recognition of the development of a particular problem.
c. It allows detection of additional symptoms.
d. It does not allow changes in therapy.
ANSWER: B
Information about the probability that a patient is experiencing or may be developing a
particular problem is obtained by comparing changes in vital signs and other signs and
symptoms.
REF: pgs. 58-59 OBJ: 2
5. When vital signs are compared with other signs and symptoms to arrive at a conclusion about
what is wrong with a patient, this is known as the:
a. review of systems.
b. differential diagnosis.
c. general clinical presentation.
d. objective assessment information.
ANSWER: B
In the field of medicine, this comparison of multiple signs and symptoms to arrive at the
patient’s diagnosis is called a differential diagnosis.
REF: pg. 59 OBJ: 2
6. If dehydration is suspected in a patient, which of the following parameters should be
monitored often?
a. Height
b. Weight
c. Fluid intake and output
d. Blood pressure
ANSWER: C
If a question arises regarding dehydration or fluid overload, fluid intake and output (I & O) and
weight may be recorded during each shift until the patient’s fluid balance is stable.
REF: pg. 59 OBJ: 2
7. A patient’s general clinical presentation indicates to the respiratory therapist (RT) that the
patient is in distress. The RT’s first step should be to:
a. call a code and stand by to assist with cardiopulmonary resuscitation (CPR).
b. do a complete physical examination to determine the precise cause of the distress.
c. evaluate the problem quickly and intervene or locate someone to assist the patient.
d. follow the procedure learned in class to introduce himself or herself to the patient
and ask the patient what is wrong.
ANSWER: C
If the patient is in distress, the priorities are to evaluate the problem in the most efficient and
REF: pg. 59 OBJ: 3
TO GET ALL CHAPTERS EMAIL ME AT>>>>>
donc8246@gmail.com
rapid way possible and to intervene or locate someone who can assist the patient.
REF: pg. 59 OBJ: 4
TO GET ALL CHAPTERS EMAIL ME AT>>>>>
donc8246@gmail.com
8. A well-written description of an RT’s initial observations is beneficial to other healthcare
providers because it:
a. shows that the RT has established rapport with the patient.
b. helps them know how to plan care and relate to the patient’s needs.
c. decreases the length of hospitalization of the patient and keeps costs down.
d. allows them to read the information quickly without confusion and
misunderstandings.
ANSWER: B
A written description of these initial observations helps others involved in the patient’s care to
know how to plan care and relate to the patient’s needs.
REF: pg. 60 OBJ: 2
9. Assessing the patient’s level of consciousness is important because it:
a. suggests the types of questions that the therapist should ask.
b. evaluates the adequacy of cerebral perfusion and oxygenation.
c. indicates whether the patient will take an active role in his or her treatment.
d. documents the patient’s mental status at the time of the physical examination.
ANSWER: B
Adequate cerebral oxygenation must be present for the patient to be conscious, alert, and well
oriented.
REF: pg. 61 OBJ: 3
10. Which of the following scales is the gold standard for assessing trends in the neurologic
function of patients who have suffered head trauma?
a. Murray scale
b. PRISM
c. APACHE
d. Glasgow scale
ANSWER: D
The Glasgow coma scale has become the gold standard for assessing trends in the neurologic
function of patients who have been sedated, have received anesthesia, have suffered head
trauma, or are near coma.
REF: pg. 61 OBJ: 3
11. The normal range for oral body temperature in most people is to ° F.
a. 97; 99.5
b. 96.7; 98.5
c. 98.7; 100.5
d. 97.7; 98.5
ANSWER: A
Normal body temperature for most persons is approximately 98.6° F (37° C), with a normal
REF: pg. 59 OBJ: 5
TO GET ALL CHAPTERS EMAIL ME AT>>>>>
donc8246@gmail.com
range from 97 to 99.5° F and daily variations of 1° to 2° F.
TO GET ALL CHAPTERS EMAIL ME AT>>>>>
donc8246@gmail.com
12. Fever is defined as an:
a. infection in the body.
b. elevation of body temperature above normal.
c. increase in the oxygen requirements of the body.
d. elevation of body temperature above normal because of disease.
ANSWER: D
When the body temperature is elevated from disease, this elevation is called fever, and the
patient is said to be febrile.
REF: pg. 62 OBJ: 3
13. What is the most common cause of fever above 102° F?
a. Aspiration pneumonitis
b. Infection
c. Blood transfusion reaction
d. Head injury
ANSWER: B
Infection is most likely to be the cause of fever when the body temperature exceeds 102° F.
REF: pg. 62 OBJ: 3
14. For every elevation in body temperature of 1° C, oxygen consumption and carbon dioxide
production increase by approximately %.
a. 2
b. 5
c. 10
d. 20
ANSWER: C
For every elevation in body temperature of 1° C, oxygen consumption and carbon dioxide
production increase by approximately 10%.
REF: pg. 62 OBJ: 3
15. Which of the following conditions causes hypothermia?
a. Damage to the hypothalamus
b. Excessive shivering and vasoconstriction
c. Exposure to high environmental temperatures
d. Invasion of the body by microorganisms that produce an infection
ANSWER: A
Hypothermia is not common but can occur in persons with a severe head injury that damages
the hypothalamus and in those suffering from exposure to cold environmental temperatures.
REF: pg. 62 OBJ: 3
16. Which of the following is not considered a typical site for temperature measurement?
a. Oral
b. Rectal
c. Axillary
TO GET ALL CHAPTERS EMAIL ME AT>>>>>
donc8246@gmail.com
d. Fingertip
ANSWER: D
Body temperature is measured most often at one of four sites: the mouth, ear, axilla, or rectum.
REF: pg. 62 OBJ: 4
17. What site is used to measure body temperature in comatose patients?
a. Oral
b. Axillary
c. Rectal
d. Ear
ANSWER: C
Rectal temperatures may be used for patients who are comatose, in intensive care, or confused.
REF: pg. 63 OBJ: 4
18. Which of the following is the method of choice for measuring temperature in neonates?
a. Oral
b. Rectal
c. Axillary
d. Tympanic
ANSWER: C
Axillary is the method of choice for neonates because it approximates their core temperature
and avoids injury to the rectal tissues.
REF: pg. 63 OBJ: 4
19. Which of the following statements is true regarding oral temperature measurements?
a. They are the most convenient to obtain and are most readily accepted by alert
adults.
b. They are useful in infants.
c. They are acceptable in orally intubated patients.
d. They are affected by delivery of oxygen by nasal cannula.
ANSWER: A
Oral temperature measurement remains a common, convenient, and acceptable method for
awake adult patients.
REF: pg. 63 OBJ: 4
20. What is the main advantage of tympanic thermometry over the classic forms of temperature
measurement?
a. It is fast, clean, and noninvasive.
b. Multiple studies have shown a high correlation with core temperature.
c. It is the preferred method of temperature measurement in the pediatric population.
d. It uses infrared emissions from the surface of the tympanic membrane and is not in
contact with skin or other tissue.
TO GET ALL CHAPTERS EMAIL ME AT>>>>>
donc8246@gmail.com
ANSWER: A
This method has the advantage of being fast, clean, and noninvasive and avoids the
embarrassment and time delays associated with the classic forms of temperature measurement.
REF: pg. 63 OBJ: 4
21. Which of the following reasons has been listed as the number one concern regarding use of
the tympanic site for body temperature measurement?
a. Speed
b. Safety
c. Accuracy
d. Cost
ANSWER: C
Concerns have been expressed about its accuracy and use in the hospital setting.
REF: pg. 63 OBJ: 4
22. The normal pulse rate range for adults is beats/min.
a. 60 to 100
b. 70 to 110
c. 80 to 120
d. 90 to 160
ANSWER: A
The normal pulse rate for adults is 60 to 100 beats/min and is regular in rhythm.
REF: pg. 64 OBJ: 5
23. A pulse rate above the normal range is the definition of which of the following terms?
a. Tachypnea
b. Arrhythmia
c. Bradycardia
d. Tachycardia
ANSWER: D
A pulse rate exceeding 100 beats/min in an adult is termed tachycardia.
REF: pg. 64 OBJ: 6
24. What is an important reason to monitor the heart rate in patients with lung disease?
a. A heart rate above or below the normal range is dangerous.
b. As the heart rate increases, the tissue’s demand for oxygen decreases.
c. The more efficiently the heart is pumping, the lower is the resting pulse rate.
d. Tachycardia is a common finding when hypoxemia is present.
ANSWER: D
When the oxygen content of arterial blood falls below normal, usually from lung disease, the
heart tries to compensate by increasing cardiac output to maintain adequate oxygen delivery to
the tissues. An increase in cardiac output is attained by an increase in heart rate in most persons.
For this reason, it is important to monitor the heart rate in patients with lung disease.
TO GET ALL CHAPTERS EMAIL ME AT>>>>>
donc8246@gmail.com
REF: pg. 64 OBJ: 5
25. Which of the following characteristics should be included in the assessment of a patient’s
pulse?
a. Age, rate, and strength
b. Rate, rhythm, and strength
c. Age, rhythm, and status
d. Strength, amplitude, and volume
ANSWER: B
The rhythm and strength of the pulse are evaluated, the pulse rate is evaluated, and then the
pulse rate is counted.
REF: pg. 64 OBJ: 5
26. Which of the following arteries is most appropriate for measuring the pulse of a patient with
very low blood pressure?
a. Radial artery
b. Popliteal artery
c. Brachial artery
d. Femoral artery
ANSWER: D
When the blood pressure is abnormally low, the more centrally located pulses, such as the
carotid pulse in the neck and femoral pulses in the groin, can be identified more easily than
the peripheral pulse.
REF: pg. 64 OBJ: 7
27. Which of the following terms is used to describe a significant decrease in pulse pressure
during spontaneous inspiration?
a. Pulsus alternans
b. Pulsus paradoxus
c. Pulsus respiratory
d. Pulsus asymmetrical
ANSWER: B
When the patient’s pulse strength decreases with spontaneous inhalation, this is referred to as
pulsus paradoxus.
REF: pg. 65 OBJ: 6
28. An alternating succession of strong and weak pulses that usually is not related to respiratory
disease is known as pulsus:
a. tardus.
b. alternans.
c. paradoxus.
d. asymmetrical.
ANSWER: B
Pulsus alternans is an alternating succession of strong and weak pulses that usually is not
related to respiratory disease.
REF: pg. 65 OBJ: 6
29. A respiratory rate of greater than breaths/min is considered abnormal at any age.
a. 60
b. 50
c. 40
d. 30
ANSWER: A
A respiratory rate of 40 breaths/min is unusual for an adult, and a rate greater than 60
breaths/min is abnormal at any age.
REF: pg. 65 OBJ: 3
30. In postoperative patients, the degree of tachypnea typically is related to the degree of:
a. anesthetic received during surgery.
b. pain medication received.
c. pneumonia.
d. atelectasis.
ANSWER: D
Tachypnea in the postoperative patient is common when significant fever develops or when the
lungs partially collapse (atelectasis) as a side effect of surgery. Atelectasis causes the lungs to
become stiffer than normal, and the patient adopts a breathing pattern that is made up of rapid
and shallow breaths, which serves as a compensatory mechanism. The degree of atelectasis
determines the degree of tachypnea in such cases.
REF: pg. 65 OBJ: 6
31. Which of the following breathing patterns describes Kussmaul breathing?
a. Normal
b. Shallow
c. Fast and shallow
d. Fast and deep
ANSWER: D
In the case of ketoacidosis resulting from uncontrolled diabetes, a fast and deep breathing
pattern known as Kussmaul breathing is often evident.
REF: pg. 65 OBJ: 6
32. Bradypnea may be caused by which of the following?
a. Hypothermia
b. Narcotic overdose
c. Head injury
d. All of the above
ANSWER: D
A slow respiratory rate, referred to as bradypnea, is uncommon but may occur in patients with
head injury or hypothermia, as a side effect of certain medications such as narcotics, and in
patients with drug overdose.
REF: pg. 65 OBJ: 6
33. Which of the following techniques is not useful in measuring the respiratory rate in patients
who are spontaneously breathing and alert?
a. Ask the patient to breathe normally.
b. Lay the hand you are using to count the patient’s pulse on the patient’s abdomen,
and watch abdominal movements.
c. Count the rate for 30 seconds and multiply by 2.
d. Pretend to be counting the pulse while counting the respiratory rate.
ANSWER: A
Never ask the patient to “breathe normally” while you are assessing the rate of respiration.
When individuals think about their breathing, they often voluntarily change their breathing
rate and pattern.
REF: pg. 66 OBJ: 8
34. The diastolic blood pressure is a reflection of:
a. the pressure exerted during left ventricular relaxation.
b. the peak pressure exerted during left ventricular contraction.
c. the difference between systolic and diastolic pressures.
d. the force exerted against the walls of the arteries as blood flows through them.
ANSWER: A
Diastolic blood pressure is the force that occurs when the heart is relaxed.
REF: pg. 66 OBJ: 8
35. Which of the following is not true about pulse pressure?
a. It normally is between 35 and 40 mm Hg.
b. It is calculated from the 12-lead electrocardiographic printout.
c. It is the difference between the systolic and diastolic pressures.
d. The peripheral pulse is difficult to detect when the pulse pressure falls to below 30
mm Hg.
ANSWER: B
Pulse pressure is the difference between the systolic and diastolic pressures. The normal pulse
pressure is 35 to 40 mm Hg. When the pulse pressure is less than 30 mm Hg, the peripheral
pulse is difficult to detect. Patients with heart failure and inadequate stroke volume usually have a
reduced pulse pressure.
REF: pg. 66 OBJ: 7
36. Which of the following organs is not at risk for disease in the presence of systemic
hypertension?
a. Heart
b. Kidneys
c. Blood vessels
d. Pancreas
ANSWER: D
Persistent pressures at the level of systemic hypertension are associated with an escalating risk
for the development of heart, vascular, and renal diseases.
REF: pg. 66 OBJ: 9
37. Which of the following is not a cause of hypotension?
a. Left ventricular failure
b. Peripheral vasodilation
c. Hypovolemia
d. Right heart failure
ANSWER: D
Hypotension may occur as the result of peripheral vasodilation, left ventricular failure, or low
blood volume.
REF: pg. 67 OBJ: 9
38. Hypotension associated with changes in posture in patients with hypovolemia is known as
hypotension.
a. persistent
b. orthostatic
c. hypovolemic
d. peristaltic
ANSWER: B
Changes in posture may produce abrupt changes in arterial blood pressure, especially in the
hypovolemic patient. Normally, when the patient moves from the supine to the sitting position,
the blood pressure changes very little, but when hypovolemia or vasodilation is present, the
blood pressure may fall significantly; this is referred to as postural (or orthostatic) hypotension.
REF: pg. 67 OBJ: 9
39. When the blood pressure is measured, the sounds that are heard through the stethoscope
between the systolic and diastolic pressures are known as sounds.
a. Meckel
b. brachial
c. Korotkoff
d. succussion
ANSWER: C
The auscultatory method of pressure measurement uses the stethoscope to listen for the sounds
produced by arterial pulse waves (Korotkoff sounds) when blood flow in the artery resumes. As
the pressure is reduced during deflation of the occluding cuff, the Korotkoff sounds change in
quality and intensity.
REF: pg. 68 OBJ: 8
40. What is the most probable cause for the significant decrease in blood pressure that occurs
during inhalation in some patients?
a. Hypovolemia
b. Variable systemic vascular resistance
c. Early left ventricular failure caused by coronary artery insufficiency
d. Negative intrathoracic pressure that diminishes blood flow out of the left ventricle
ANSWER: D
This decrease in systolic blood pressure is more significant during a forced maximal inhalation.
When the systolic pressure drops by more than 10 mm Hg during inhalation at rest, a definite
abnormality exists; this is termed paradoxic pulse. Paradoxic pulse, also called pulsus paradoxus,
occurs in various circulatory and respiratory conditions such as asthma and cardiac tamponade.
The most probable mechanism responsible for this fluctuation in blood pressure centers around
the negative intrathoracic pressure created by the respiratory muscles during inhalation.
REF: pg. 70 OBJ: 11
41. Which of the following conditions is not associated with the presence of pulsus paradoxus?
a. Hypervolemia
b. Cardiac tamponade
c. Constrictive pericarditis
d. Asthma
ANSWER: A
Pulsus paradoxus is commonly seen in patients with restrictions around the heart, such as
cardiac tamponade, constrictive pericarditis, or restrictive cardiomyopathy. It also may occur in
patients with severe pulmonary diseases, such as acute asthma.
REF: pg. 70 OBJ: 11
42. A 42-year-old male patient with a history of poorly controlled asthma is seen in the emergency
department by the respiratory therapist (RT). The patient is seated in a chair and clearly is in
extreme respiratory distress, with inspiratory and expiratory wheezing throughout. He reports
that he has had a fever for the past 2 days and has eaten and drunk very little. He also reports
some chest pain, and his blood pressure is 95/55 mm Hg. At one point, the patient stands to
move from the chair to the stretcher and has an episode of syncope. The differential diagnosis
for this patient should include all of the following except:
a. Myocardial infarction (MI).
b. Acute asthma exacerbation.
c. Dehydration.
d. Cerebrovascular accident (CVA; stroke).
ANSWER: D
The chest pain, although minor, requires evaluation for MI. The wheezing and respiratory
distress strongly suggest asthma exacerbation, and the combination of syncope, restricted fluid
restriction, and low blood pressure strongly suggest hypovolemia. No signs or symptoms of
CVA are present.
REF: pg. 70 OBJ: 9
43. An RT attempts to take the blood pressure of the patient in question 42. The RT notes that the
pulse intensity determined from palpation of the radial artery decreases during inspiration. The
most likely explanation for this would be:
a. an (MI).
b. an acute asthma exacerbation.
c. dehydration.
d. a cerebrovascular accident (CVA; stroke).
ANSWER: B
Because of the high airway resistance caused by the asthma exacerbation, the intrathoracic
pressure decreases abnormally during patient inspiration. This results in decreased cardiac filling,
decreased blood pressure, and decreased pulse intensity during inspiration. This is an example of
pulsus paradoxus.
REF: pg. 70 OBJ: 11
44. An RT is called to the emergency department on New Year ’s Eve to assist in the treatment of a
5-year-old child pulled from under the ice of a local pond. When the child arrives, he has been
intubated and has a weak, irregularly irregular pulse of 55 beats/min, no spontaneous
respiratory effort, and a body temperature of 30o
C. The normal practice in this hospital is to
use heat–moisture exchangers (HMEs) for all mechanically ventilated patients. Which of the
following should the therapist suggest for the ventilator setup?
a. Greater than normal rate and tidal volume
b. Higher than usual inspiratory flow rate
c. Heated humidity in place of the HME
d. All of the above
ANSWER: C
This patient is clearly hypothermic; therefore, along with other warming measures, heated,
humidified air will significantly aid in raising the patient’s body temperature. Normally,
ventilator settings for these patients require lower minute ventilation than usual. An increased
rate and tidal volume would result in increased minute ventilation, whereas an increased
inspiratory flow rate would result in an elevated peak inspiratory pressure.
REF: pg. 62 OBJ: 3
45. An RT has just finished administering an aerosol treatment by a small-volume nebulizer to a
patient who has a serious case of pneumonia. For the past 24 hours, the patient has had a fever
of 39 to 40o
C. Just as the therapist is finishing, a nursing aide comes in to record the vital signs.
She uses an electronic thermometer to record an oral temperature of 36.5o
C and comments to
the patient, “Well, Mr. Jones, you must be getting better because your fever is completely gone.”
The therapist should:
a. also verbally encourage the patient by saying that he is getting better.
b. totally ignore the situation and continue with the posttreatment assessment.
c. complete the treatment and assessment, record the results in the patient’s chart, then
go on to the next patient.
d. go to the nursing aide after she leaves the room and tell her that the temperature she
recorded is probably not accurate and should be repeated.
ANSWER: D
A small-volume nebulizer produces an aerosol mist that may temporarily cool the inside of the
mouth. Hence, an oral temperature should not be measured for at least 10 to 15 minutes
following the completion of the nebulizer treatment.
REF: pg. 63 OBJ: 3
46. An RT is taking the blood pressure in the arm of a 12-year-old girl. The only
sphygmomanometer available has a cuff in which the ends overlap slightly when it is inflated.
The pressure that the RT records:
a. should be accurate if the measurement is otherwise done properly.
b. is likely to be in error, showing a value that is too low.
c. is likely to be in error, showing a value that is too high.
d. will have a value that cannot be predicted to be either too high or too low.
ANSWER: A
When measuring the blood pressure in a child, it is permissible to use a cuff where the ends
overlap slightly.
REF: pg. 68 OBJ: 10
Chapter5: Fundamentals of Physical Examination
Test Bank
MULTIPLE CHOICE
1. All of the following are components of the physical examination except:
a. inspection.
b. auscultation.
c. manipulation.
d. percussion.
ANSWER: C
The four components of the physical examination are inspection, palpation, percussion, and
auscultation.
REF: pg. 75 OBJ: 1
2. A review of the patient’s history of present illness and past clinical history prior to a physical
examination is helpful in that it:
a. gives insight into the expected findings during the examination.
b. suggests examination techniques to emphasize.
c. helps establish a rapport with the patient.
d. all of the above.
ANSWER: D
A review of the history helps direct the course of the physical examination, allows the medical
care team to begin to establish rapport with the patient, and may suggest the success or failure of
past treatments.
REF: pg. 76 OBJ: 2
3. What problem is associated with cyanosis of the oral mucosa?
a. Low cardiac output
b. Reduced arterial oxygenation
c. Hyperventilation
d. Hypoventilation
ANSWER: B
Cyanosis of the oral mucosa is a sign of central cyanosis that occurs only with severe
hypoxemia.
REF: pg. 76 OBJ: 3
4. What problem is associated with nasal flaring?
a. Hypoxemia
b. Increased lung compliance
c. Increased work of breathing
d. Increased dead space ventilation
ANSWER: C
Nasal flaring most often occurs in infants and children who are working very hard to breathe.
Nasal flaring usually occurs when the lungs stiffen (e.g., with pneumonia) or when the airways
narrow (e.g., with asthma).
REF: pg. 76 OBJ: 3
5. Which of the following is not associated with dilated and unresponsive pupils?
a. Opiates
b. Atropine
c. Brain death
d. Catecholamines
ANSWER: A
Atropine, brain hypoxia, and catecholamines can cause the pupils to become fixed and
unresponsive to light. Opiates have no such effect.
REF: pg. 76 OBJ: 3
6. Which of the following terms is used to describe constriction of the pupil of the eye?
a. Lytosis
b. Miosis
c. Mydriasis
d. Nystagmus
ANSWER: B
Miosis refers to constriction of the pupils.
REF: pg. 76 OBJ: 3
7. Which of the following terms is used to describe drooping of the eyelids?
a. Ptosis
b. Miosis
c. Diplopia
d. Chondrosis
ANSWER: A
Ptosis is defined as drooping of the eyelids.
REF: pg. 77 OBJ: 3
8. Which of the following physical examination findings is associated with myasthenia gravis?
a. Ptosis
b. Miosis
c. Mydriasis
d. Hepatomegaly
ANSWER: A
Patients with myasthenia gravis often develop ptosis as part of the clinical picture.
REF: pg. 77 OBJ: 3
9. Which of the following terms is used to describe double vision?
a. Miosis
b. Diplopia
c. Nystagmus
d. None of the above
ANSWER: B
Diplopia refers to double vision.
REF: pg. 77 OBJ: 3
10. Which of the following is most likely to cause tracheal deviation?
a. Left upper lobe pneumonia
b. Right upper lobe pneumothorax
c. Left lower lobe pleural effusion
d. Right lower lobe pleural effusion
ANSWER: B
Upper lobe changes are more likely to influence tracheal position than are lower lobe
problems. Pneumothorax usually causes a volume change in the upper lobe and alters the
position of the trachea.
REF: pg. 77 OBJ: 3
11. Which of the following is the most common cause of jugular venous distention?
a. Auto-positive end-expiratory pressure (auto-PEEP)
b. Pneumonia
c. Pneumothorax
d. Right heart failure
ANSWER: D
Right heart failure causes a backup of venous blood into the venous system. This causes the
neck veins to distend.
REF: pg. 78 OBJ: 3
12. At what angle should the head of the bed be elevated to assess jugular vein pressure (JVP)?
a. 15 degrees
b. 30 degrees
c. 45 degrees
d. 90 degrees
ANSWER: C
The current standardized procedure for checking the JVP at the bedside is to elevate the head of
the bed to 45 degrees. If the head of the bed were placed at 90 degrees, gravity would make it
difficult to see the JVP. If the head of the bed were placed at less than 45 degrees, venous blood
would tend to pool in the neck, which would cause most patients to appear to have JVP.
REF: pg. 77 OBJ: 4
13. In a patient’s chart, the physician has documented finding tender lymph nodes in the neck
region during the physical examination. This problem is consistent with:
a. respiratory infection.
b. human immunodeficiency virus (HIV) infection.
c. lymphoma.
d. lung cancer.
ANSWER: A
Tender lymph nodes often are associated with infection.
REF: pg. 78 OBJ: None
14. The horizontal fissure that separates the right upper lobe from the right middle lobe begins at
what rib in the midsternal line?
a. Rib 2
b. Rib 4
c. Rib 6
d. Rib 8
ANSWER: B
The horizontal fissure is located at the fourth rib at the midsternal line.
REF: pg. 79 OBJ: 5
15. At what point on the posterior chest wall can the inferior border of the lung normally be
found?
a. Between T4 and T6
b. Between T6 and T8
c. Between T9 and T12
d. None of the above
ANSWER: C
Normally, the inferior border of the lung is located between T9 and T12 on the posterior chest.
REF: pg. 80 OBJ: 5
16. Which of the following terms describes an abnormal lateral curvature of the spine?
a. Kyphosis
b. Scoliosis
c. Anhidrosis
d. Myelosis
ANSWER: B
The term for lateral curvature of the spine is scoliosis. Kyphosis refers to an abnormal
anteroposterior curvature of the spine.
REF: pg. 80 OBJ: 6
17. Which of the following disorders is associated with a barrel chest?
a. Chronic bronchitis
b. Pneumonia
c. Emphysema
d. Chest trauma
ANS: C
Patients with emphysema lose their lung recoil. This allows the ribs to straighten somewhat,
causing the patient’s anteroposterior diameter to increase significantly.
REF: pg. 80 OBJ: 6
18. Which of the following patterns of breathing is associated with a loss in lung volume?
a. Slow and deep
b. Rapid and deep
c. Slow and shallow
d. Rapid and shallow
ANSWER: D
A loss of lung volume causes lung compliance to decrease. This makes it more difficult for the
patient to breathe at normal tidal volumes. As a result, patients breathe with smaller volumes but
at a faster rate.
REF: pg. 80 OBJ: 8
19. What disease is most likely to cause a prolonged expiratory time?
a. Asthma
b. Pneumonia
c. Pneumothorax
d. Pulmonary fibrosis
ANSWER: A
Asthma causes the intrathoracic airways to become narrow, which makes exhalation slow and
prolonged.
REF: pg. 80 OBJ: 8
20. Which of the following diseases is most likely to cause a prolonged inspiratory time?
a. Chronic obstructive pulmonary disease (COPD)
b. Asthma
c. Epiglottitis
d. Acute bronchitis
ANSWER: C
Epiglottitis causes narrowing of the upper airway. This causes a prolonged inspiratory time
because the narrowed upper airway slows inspiratory flow more than expiratory flow.
REF: pg. 82 OBJ: 8
21. Which of the following terms describes the sinking inward of the skin overlying the rib cage
with each inspiratory effort?
a. Ataxia
b. Bulging
c. Retractions
d. Mydriasis
ANS: C

More Related Content

Similar to TEST BANK for Wilkins’ Clinical Assessment in Respiratory Care, 9th Edition by Albert.pdf

TEST BANK For Advanced Practice Nursing in the Care of Older Adults, 2nd Edit...
TEST BANK For Advanced Practice Nursing in the Care of Older Adults, 2nd Edit...TEST BANK For Advanced Practice Nursing in the Care of Older Adults, 2nd Edit...
TEST BANK For Advanced Practice Nursing in the Care of Older Adults, 2nd Edit...
kevinkariuki227
 
ME 325 Thermodynamics 2Homework assignment 6Note For probl.docx
ME 325 Thermodynamics 2Homework assignment 6Note For probl.docxME 325 Thermodynamics 2Homework assignment 6Note For probl.docx
ME 325 Thermodynamics 2Homework assignment 6Note For probl.docx
roushhsiu
 
Free ebooks for all courses
Free ebooks for all coursesFree ebooks for all courses
Free ebooks for all courses
Edhole.com
 
Mca admission in india
Mca admission in indiaMca admission in india
Mca admission in india
Edhole.com
 
Mca admission in india
Mca admission in indiaMca admission in india
Mca admission in indiaEdhole.com
 
Clinical Nursing Skills and Techniques 9th Edition Perry Test Bank
Clinical Nursing Skills and Techniques 9th Edition Perry Test BankClinical Nursing Skills and Techniques 9th Edition Perry Test Bank
Clinical Nursing Skills and Techniques 9th Edition Perry Test Bank
siqeqyryxi
 
CNA Practice Exam 2013
CNA Practice Exam 2013CNA Practice Exam 2013
CNA Practice Exam 2013
Baodong Li
 
Module 02 ContentAPA formatTop of FormPurposeTo use critic.docx
Module 02 ContentAPA formatTop of FormPurposeTo use critic.docxModule 02 ContentAPA formatTop of FormPurposeTo use critic.docx
Module 02 ContentAPA formatTop of FormPurposeTo use critic.docx
kendalfarrier
 
Krok 2 - 2014 (Hygiene)
Krok 2 - 2014 (Hygiene)Krok 2 - 2014 (Hygiene)
Krok 2 - 2014 (Hygiene)
Eneutron
 
Test bank for critical care nursing a holistic approach 11th edition morton f...
Test bank for critical care nursing a holistic approach 11th edition morton f...Test bank for critical care nursing a holistic approach 11th edition morton f...
Test bank for critical care nursing a holistic approach 11th edition morton f...
robinsonayot
 
Community Health Nursing MCQs Purbanchal University Nepal
Community Health Nursing MCQs Purbanchal University NepalCommunity Health Nursing MCQs Purbanchal University Nepal
Community Health Nursing MCQs Purbanchal University Nepal
Ram Lal Mahatara
 
104777007 guillain-barre-syndrome-case-study-group
104777007 guillain-barre-syndrome-case-study-group104777007 guillain-barre-syndrome-case-study-group
104777007 guillain-barre-syndrome-case-study-group
homeworkping7
 
TEST BANK for The Nursing Assistant Acute, Subacute, and Long-Term Care, 6th ...
TEST BANK for The Nursing Assistant Acute, Subacute, and Long-Term Care, 6th ...TEST BANK for The Nursing Assistant Acute, Subacute, and Long-Term Care, 6th ...
TEST BANK for The Nursing Assistant Acute, Subacute, and Long-Term Care, 6th ...
kevinkariuki227
 
Fundamental of nursing review questions
Fundamental of nursing review questionsFundamental of nursing review questions
Fundamental of nursing review questions
Nursing School Questions
 
9 questionsbook
9 questionsbook9 questionsbook
9 questionsbook
AngelaDanielleTan
 
HIT1443 LEIHP4e Ch10
HIT1443 LEIHP4e Ch10HIT1443 LEIHP4e Ch10
HIT1443 LEIHP4e Ch10
JLynn Jen Smith
 
Test Bank For Canadian Physical Examination and Health Assessment 3rd Edition...
Test Bank For Canadian Physical Examination and Health Assessment 3rd Edition...Test Bank For Canadian Physical Examination and Health Assessment 3rd Edition...
Test Bank For Canadian Physical Examination and Health Assessment 3rd Edition...
nursing premium
 
scribd.vpdfs.com_recalls_1_ratio_day_1_2.pdf.pdf
scribd.vpdfs.com_recalls_1_ratio_day_1_2.pdf.pdfscribd.vpdfs.com_recalls_1_ratio_day_1_2.pdf.pdf
scribd.vpdfs.com_recalls_1_ratio_day_1_2.pdf.pdf
ShillaMaeBalance1
 
Electronic portfolio
Electronic portfolioElectronic portfolio
Electronic portfolio
Tyler Gutschenritter
 

Similar to TEST BANK for Wilkins’ Clinical Assessment in Respiratory Care, 9th Edition by Albert.pdf (20)

TEST BANK For Advanced Practice Nursing in the Care of Older Adults, 2nd Edit...
TEST BANK For Advanced Practice Nursing in the Care of Older Adults, 2nd Edit...TEST BANK For Advanced Practice Nursing in the Care of Older Adults, 2nd Edit...
TEST BANK For Advanced Practice Nursing in the Care of Older Adults, 2nd Edit...
 
ME 325 Thermodynamics 2Homework assignment 6Note For probl.docx
ME 325 Thermodynamics 2Homework assignment 6Note For probl.docxME 325 Thermodynamics 2Homework assignment 6Note For probl.docx
ME 325 Thermodynamics 2Homework assignment 6Note For probl.docx
 
Free ebooks for all courses
Free ebooks for all coursesFree ebooks for all courses
Free ebooks for all courses
 
Mca admission in india
Mca admission in indiaMca admission in india
Mca admission in india
 
Mca admission in india
Mca admission in indiaMca admission in india
Mca admission in india
 
Clinical Nursing Skills and Techniques 9th Edition Perry Test Bank
Clinical Nursing Skills and Techniques 9th Edition Perry Test BankClinical Nursing Skills and Techniques 9th Edition Perry Test Bank
Clinical Nursing Skills and Techniques 9th Edition Perry Test Bank
 
CNA Practice Exam 2013
CNA Practice Exam 2013CNA Practice Exam 2013
CNA Practice Exam 2013
 
Module 02 ContentAPA formatTop of FormPurposeTo use critic.docx
Module 02 ContentAPA formatTop of FormPurposeTo use critic.docxModule 02 ContentAPA formatTop of FormPurposeTo use critic.docx
Module 02 ContentAPA formatTop of FormPurposeTo use critic.docx
 
Krok 2 - 2014 (Hygiene)
Krok 2 - 2014 (Hygiene)Krok 2 - 2014 (Hygiene)
Krok 2 - 2014 (Hygiene)
 
Test bank for critical care nursing a holistic approach 11th edition morton f...
Test bank for critical care nursing a holistic approach 11th edition morton f...Test bank for critical care nursing a holistic approach 11th edition morton f...
Test bank for critical care nursing a holistic approach 11th edition morton f...
 
Community Health Nursing MCQs Purbanchal University Nepal
Community Health Nursing MCQs Purbanchal University NepalCommunity Health Nursing MCQs Purbanchal University Nepal
Community Health Nursing MCQs Purbanchal University Nepal
 
104777007 guillain-barre-syndrome-case-study-group
104777007 guillain-barre-syndrome-case-study-group104777007 guillain-barre-syndrome-case-study-group
104777007 guillain-barre-syndrome-case-study-group
 
TEST BANK for The Nursing Assistant Acute, Subacute, and Long-Term Care, 6th ...
TEST BANK for The Nursing Assistant Acute, Subacute, and Long-Term Care, 6th ...TEST BANK for The Nursing Assistant Acute, Subacute, and Long-Term Care, 6th ...
TEST BANK for The Nursing Assistant Acute, Subacute, and Long-Term Care, 6th ...
 
Fundamental of nursing review questions
Fundamental of nursing review questionsFundamental of nursing review questions
Fundamental of nursing review questions
 
9 questionsbook
9 questionsbook9 questionsbook
9 questionsbook
 
HIT1443 LEIHP4e Ch10
HIT1443 LEIHP4e Ch10HIT1443 LEIHP4e Ch10
HIT1443 LEIHP4e Ch10
 
Test Bank For Canadian Physical Examination and Health Assessment 3rd Edition...
Test Bank For Canadian Physical Examination and Health Assessment 3rd Edition...Test Bank For Canadian Physical Examination and Health Assessment 3rd Edition...
Test Bank For Canadian Physical Examination and Health Assessment 3rd Edition...
 
scribd.vpdfs.com_recalls_1_ratio_day_1_2.pdf.pdf
scribd.vpdfs.com_recalls_1_ratio_day_1_2.pdf.pdfscribd.vpdfs.com_recalls_1_ratio_day_1_2.pdf.pdf
scribd.vpdfs.com_recalls_1_ratio_day_1_2.pdf.pdf
 
Electronic portfolio
Electronic portfolioElectronic portfolio
Electronic portfolio
 
Drill 5
Drill 5Drill 5
Drill 5
 

More from Donc Test

Libby/Libby/Hodge Financial Accounting 11th Edition TEST BANK & SOLUTION MANU...
Libby/Libby/Hodge Financial Accounting 11th Edition TEST BANK & SOLUTION MANU...Libby/Libby/Hodge Financial Accounting 11th Edition TEST BANK & SOLUTION MANU...
Libby/Libby/Hodge Financial Accounting 11th Edition TEST BANK & SOLUTION MANU...
Donc Test
 
Solution Manual For Financial Accounting, 8th Canadian Edition 2024, by Libby...
Solution Manual For Financial Accounting, 8th Canadian Edition 2024, by Libby...Solution Manual For Financial Accounting, 8th Canadian Edition 2024, by Libby...
Solution Manual For Financial Accounting, 8th Canadian Edition 2024, by Libby...
Donc Test
 
TEST BANK For Community and Public Health Nursing: Evidence for Practice, 4th...
TEST BANK For Community and Public Health Nursing: Evidence for Practice, 4th...TEST BANK For Community and Public Health Nursing: Evidence for Practice, 4th...
TEST BANK For Community and Public Health Nursing: Evidence for Practice, 4th...
Donc Test
 
TEST BANK For Community and Public Health Nursing: Evidence for Practice, 3rd...
TEST BANK For Community and Public Health Nursing: Evidence for Practice, 3rd...TEST BANK For Community and Public Health Nursing: Evidence for Practice, 3rd...
TEST BANK For Community and Public Health Nursing: Evidence for Practice, 3rd...
Donc Test
 
TEST BANK For Community Health Nursing A Canadian Perspective, 5th Edition by...
TEST BANK For Community Health Nursing A Canadian Perspective, 5th Edition by...TEST BANK For Community Health Nursing A Canadian Perspective, 5th Edition by...
TEST BANK For Community Health Nursing A Canadian Perspective, 5th Edition by...
Donc Test
 
TEST BANK For CURRENT Diagnosis & Treatment Pediatrics, 26th Edition by Maya ...
TEST BANK For CURRENT Diagnosis & Treatment Pediatrics, 26th Edition by Maya ...TEST BANK For CURRENT Diagnosis & Treatment Pediatrics, 26th Edition by Maya ...
TEST BANK For CURRENT Diagnosis & Treatment Pediatrics, 26th Edition by Maya ...
Donc Test
 
TEST BANK For Current Medical Diagnosis And Treatment 2024, 63rd Edition By M...
TEST BANK For Current Medical Diagnosis And Treatment 2024, 63rd Edition By M...TEST BANK For Current Medical Diagnosis And Treatment 2024, 63rd Edition By M...
TEST BANK For Current Medical Diagnosis And Treatment 2024, 63rd Edition By M...
Donc Test
 
TEST BANK For Davis Advantage for Pathophysiology Introductory Concepts and C...
TEST BANK For Davis Advantage for Pathophysiology Introductory Concepts and C...TEST BANK For Davis Advantage for Pathophysiology Introductory Concepts and C...
TEST BANK For Davis Advantage for Pathophysiology Introductory Concepts and C...
Donc Test
 
TEST BANK For Discovering the Life Span, 5th Edition Robert S. Feldman, Verif...
TEST BANK For Discovering the Life Span, 5th Edition Robert S. Feldman, Verif...TEST BANK For Discovering the Life Span, 5th Edition Robert S. Feldman, Verif...
TEST BANK For Discovering the Life Span, 5th Edition Robert S. Feldman, Verif...
Donc Test
 
Test Bank - Karch Focus on Nursing Pharmacology 9th Edition by Rebecca Tucker...
Test Bank - Karch Focus on Nursing Pharmacology 9th Edition by Rebecca Tucker...Test Bank - Karch Focus on Nursing Pharmacology 9th Edition by Rebecca Tucker...
Test Bank - Karch Focus on Nursing Pharmacology 9th Edition by Rebecca Tucker...
Donc Test
 
Test Bank For Clinical Nursing Skills and Techniques 10th Edition (1).pdf
Test Bank For Clinical Nursing Skills and Techniques 10th Edition (1).pdfTest Bank For Clinical Nursing Skills and Techniques 10th Edition (1).pdf
Test Bank For Clinical Nursing Skills and Techniques 10th Edition (1).pdf
Donc Test
 
Test Bank for Anatomy of Oriented Structure 8th edition.pdf
Test Bank for Anatomy of Oriented Structure 8th edition.pdfTest Bank for Anatomy of Oriented Structure 8th edition.pdf
Test Bank for Anatomy of Oriented Structure 8th edition.pdf
Donc Test
 
TEST BANK Essentials of dental radiography 9th edition by Evelyn Thomson, Orl...
TEST BANK Essentials of dental radiography 9th edition by Evelyn Thomson, Orl...TEST BANK Essentials of dental radiography 9th edition by Evelyn Thomson, Orl...
TEST BANK Essentials of dental radiography 9th edition by Evelyn Thomson, Orl...
Donc Test
 
Test bank clinical nursing skills a concept based approach 4e pearson educati...
Test bank clinical nursing skills a concept based approach 4e pearson educati...Test bank clinical nursing skills a concept based approach 4e pearson educati...
Test bank clinical nursing skills a concept based approach 4e pearson educati...
Donc Test
 
TEST BANK For, Information Technology Project Management 9th Edition Kathy Sc...
TEST BANK For, Information Technology Project Management 9th Edition Kathy Sc...TEST BANK For, Information Technology Project Management 9th Edition Kathy Sc...
TEST BANK For, Information Technology Project Management 9th Edition Kathy Sc...
Donc Test
 
TEST BANK For Wong's Nursing Care of Infants and Children, 11th Edition by Ma...
TEST BANK For Wong's Nursing Care of Infants and Children, 11th Edition by Ma...TEST BANK For Wong's Nursing Care of Infants and Children, 11th Edition by Ma...
TEST BANK For Wong's Nursing Care of Infants and Children, 11th Edition by Ma...
Donc Test
 
TEST BANK For Understanding the Essentials of Critical Care Nursing, 3rd Edit...
TEST BANK For Understanding the Essentials of Critical Care Nursing, 3rd Edit...TEST BANK For Understanding the Essentials of Critical Care Nursing, 3rd Edit...
TEST BANK For Understanding the Essentials of Critical Care Nursing, 3rd Edit...
Donc Test
 
TEST BANK For Timby's Introductory Medical-Surgical Nursing, 13th Edition by ...
TEST BANK For Timby's Introductory Medical-Surgical Nursing, 13th Edition by ...TEST BANK For Timby's Introductory Medical-Surgical Nursing, 13th Edition by ...
TEST BANK For Timby's Introductory Medical-Surgical Nursing, 13th Edition by ...
Donc Test
 
TEST BANK For Paramedic Care - Principles and Practice, 6th Edition, Volume 1...
TEST BANK For Paramedic Care - Principles and Practice, 6th Edition, Volume 1...TEST BANK For Paramedic Care - Principles and Practice, 6th Edition, Volume 1...
TEST BANK For Paramedic Care - Principles and Practice, 6th Edition, Volume 1...
Donc Test
 
TEST BANK For Paramedic Care - Principles and Practice, 6th Edition, Volume 1...
TEST BANK For Paramedic Care - Principles and Practice, 6th Edition, Volume 1...TEST BANK For Paramedic Care - Principles and Practice, 6th Edition, Volume 1...
TEST BANK For Paramedic Care - Principles and Practice, 6th Edition, Volume 1...
Donc Test
 

More from Donc Test (20)

Libby/Libby/Hodge Financial Accounting 11th Edition TEST BANK & SOLUTION MANU...
Libby/Libby/Hodge Financial Accounting 11th Edition TEST BANK & SOLUTION MANU...Libby/Libby/Hodge Financial Accounting 11th Edition TEST BANK & SOLUTION MANU...
Libby/Libby/Hodge Financial Accounting 11th Edition TEST BANK & SOLUTION MANU...
 
Solution Manual For Financial Accounting, 8th Canadian Edition 2024, by Libby...
Solution Manual For Financial Accounting, 8th Canadian Edition 2024, by Libby...Solution Manual For Financial Accounting, 8th Canadian Edition 2024, by Libby...
Solution Manual For Financial Accounting, 8th Canadian Edition 2024, by Libby...
 
TEST BANK For Community and Public Health Nursing: Evidence for Practice, 4th...
TEST BANK For Community and Public Health Nursing: Evidence for Practice, 4th...TEST BANK For Community and Public Health Nursing: Evidence for Practice, 4th...
TEST BANK For Community and Public Health Nursing: Evidence for Practice, 4th...
 
TEST BANK For Community and Public Health Nursing: Evidence for Practice, 3rd...
TEST BANK For Community and Public Health Nursing: Evidence for Practice, 3rd...TEST BANK For Community and Public Health Nursing: Evidence for Practice, 3rd...
TEST BANK For Community and Public Health Nursing: Evidence for Practice, 3rd...
 
TEST BANK For Community Health Nursing A Canadian Perspective, 5th Edition by...
TEST BANK For Community Health Nursing A Canadian Perspective, 5th Edition by...TEST BANK For Community Health Nursing A Canadian Perspective, 5th Edition by...
TEST BANK For Community Health Nursing A Canadian Perspective, 5th Edition by...
 
TEST BANK For CURRENT Diagnosis & Treatment Pediatrics, 26th Edition by Maya ...
TEST BANK For CURRENT Diagnosis & Treatment Pediatrics, 26th Edition by Maya ...TEST BANK For CURRENT Diagnosis & Treatment Pediatrics, 26th Edition by Maya ...
TEST BANK For CURRENT Diagnosis & Treatment Pediatrics, 26th Edition by Maya ...
 
TEST BANK For Current Medical Diagnosis And Treatment 2024, 63rd Edition By M...
TEST BANK For Current Medical Diagnosis And Treatment 2024, 63rd Edition By M...TEST BANK For Current Medical Diagnosis And Treatment 2024, 63rd Edition By M...
TEST BANK For Current Medical Diagnosis And Treatment 2024, 63rd Edition By M...
 
TEST BANK For Davis Advantage for Pathophysiology Introductory Concepts and C...
TEST BANK For Davis Advantage for Pathophysiology Introductory Concepts and C...TEST BANK For Davis Advantage for Pathophysiology Introductory Concepts and C...
TEST BANK For Davis Advantage for Pathophysiology Introductory Concepts and C...
 
TEST BANK For Discovering the Life Span, 5th Edition Robert S. Feldman, Verif...
TEST BANK For Discovering the Life Span, 5th Edition Robert S. Feldman, Verif...TEST BANK For Discovering the Life Span, 5th Edition Robert S. Feldman, Verif...
TEST BANK For Discovering the Life Span, 5th Edition Robert S. Feldman, Verif...
 
Test Bank - Karch Focus on Nursing Pharmacology 9th Edition by Rebecca Tucker...
Test Bank - Karch Focus on Nursing Pharmacology 9th Edition by Rebecca Tucker...Test Bank - Karch Focus on Nursing Pharmacology 9th Edition by Rebecca Tucker...
Test Bank - Karch Focus on Nursing Pharmacology 9th Edition by Rebecca Tucker...
 
Test Bank For Clinical Nursing Skills and Techniques 10th Edition (1).pdf
Test Bank For Clinical Nursing Skills and Techniques 10th Edition (1).pdfTest Bank For Clinical Nursing Skills and Techniques 10th Edition (1).pdf
Test Bank For Clinical Nursing Skills and Techniques 10th Edition (1).pdf
 
Test Bank for Anatomy of Oriented Structure 8th edition.pdf
Test Bank for Anatomy of Oriented Structure 8th edition.pdfTest Bank for Anatomy of Oriented Structure 8th edition.pdf
Test Bank for Anatomy of Oriented Structure 8th edition.pdf
 
TEST BANK Essentials of dental radiography 9th edition by Evelyn Thomson, Orl...
TEST BANK Essentials of dental radiography 9th edition by Evelyn Thomson, Orl...TEST BANK Essentials of dental radiography 9th edition by Evelyn Thomson, Orl...
TEST BANK Essentials of dental radiography 9th edition by Evelyn Thomson, Orl...
 
Test bank clinical nursing skills a concept based approach 4e pearson educati...
Test bank clinical nursing skills a concept based approach 4e pearson educati...Test bank clinical nursing skills a concept based approach 4e pearson educati...
Test bank clinical nursing skills a concept based approach 4e pearson educati...
 
TEST BANK For, Information Technology Project Management 9th Edition Kathy Sc...
TEST BANK For, Information Technology Project Management 9th Edition Kathy Sc...TEST BANK For, Information Technology Project Management 9th Edition Kathy Sc...
TEST BANK For, Information Technology Project Management 9th Edition Kathy Sc...
 
TEST BANK For Wong's Nursing Care of Infants and Children, 11th Edition by Ma...
TEST BANK For Wong's Nursing Care of Infants and Children, 11th Edition by Ma...TEST BANK For Wong's Nursing Care of Infants and Children, 11th Edition by Ma...
TEST BANK For Wong's Nursing Care of Infants and Children, 11th Edition by Ma...
 
TEST BANK For Understanding the Essentials of Critical Care Nursing, 3rd Edit...
TEST BANK For Understanding the Essentials of Critical Care Nursing, 3rd Edit...TEST BANK For Understanding the Essentials of Critical Care Nursing, 3rd Edit...
TEST BANK For Understanding the Essentials of Critical Care Nursing, 3rd Edit...
 
TEST BANK For Timby's Introductory Medical-Surgical Nursing, 13th Edition by ...
TEST BANK For Timby's Introductory Medical-Surgical Nursing, 13th Edition by ...TEST BANK For Timby's Introductory Medical-Surgical Nursing, 13th Edition by ...
TEST BANK For Timby's Introductory Medical-Surgical Nursing, 13th Edition by ...
 
TEST BANK For Paramedic Care - Principles and Practice, 6th Edition, Volume 1...
TEST BANK For Paramedic Care - Principles and Practice, 6th Edition, Volume 1...TEST BANK For Paramedic Care - Principles and Practice, 6th Edition, Volume 1...
TEST BANK For Paramedic Care - Principles and Practice, 6th Edition, Volume 1...
 
TEST BANK For Paramedic Care - Principles and Practice, 6th Edition, Volume 1...
TEST BANK For Paramedic Care - Principles and Practice, 6th Edition, Volume 1...TEST BANK For Paramedic Care - Principles and Practice, 6th Edition, Volume 1...
TEST BANK For Paramedic Care - Principles and Practice, 6th Edition, Volume 1...
 

Recently uploaded

Superficial & Deep Fascia of the NECK.pptx
Superficial & Deep Fascia of the NECK.pptxSuperficial & Deep Fascia of the NECK.pptx
Superficial & Deep Fascia of the NECK.pptx
Dr. Rabia Inam Gandapore
 
Pharynx and Clinical Correlations BY Dr.Rabia Inam Gandapore.pptx
Pharynx and Clinical Correlations BY Dr.Rabia Inam Gandapore.pptxPharynx and Clinical Correlations BY Dr.Rabia Inam Gandapore.pptx
Pharynx and Clinical Correlations BY Dr.Rabia Inam Gandapore.pptx
Dr. Rabia Inam Gandapore
 
ARTHROLOGY PPT NCISM SYLLABUS AYURVEDA STUDENTS
ARTHROLOGY PPT NCISM SYLLABUS AYURVEDA STUDENTSARTHROLOGY PPT NCISM SYLLABUS AYURVEDA STUDENTS
ARTHROLOGY PPT NCISM SYLLABUS AYURVEDA STUDENTS
Dr. Vinay Pareek
 
Ozempic: Preoperative Management of Patients on GLP-1 Receptor Agonists
Ozempic: Preoperative Management of Patients on GLP-1 Receptor Agonists  Ozempic: Preoperative Management of Patients on GLP-1 Receptor Agonists
Ozempic: Preoperative Management of Patients on GLP-1 Receptor Agonists
Saeid Safari
 
Pharma Pcd Franchise in Jharkhand - Yodley Lifesciences
Pharma Pcd Franchise in Jharkhand - Yodley LifesciencesPharma Pcd Franchise in Jharkhand - Yodley Lifesciences
Pharma Pcd Franchise in Jharkhand - Yodley Lifesciences
Yodley Lifesciences
 
Flu Vaccine Alert in Bangalore Karnataka
Flu Vaccine Alert in Bangalore KarnatakaFlu Vaccine Alert in Bangalore Karnataka
Flu Vaccine Alert in Bangalore Karnataka
addon Scans
 
Triangles of Neck and Clinical Correlation by Dr. RIG.pptx
Triangles of Neck and Clinical Correlation by Dr. RIG.pptxTriangles of Neck and Clinical Correlation by Dr. RIG.pptx
Triangles of Neck and Clinical Correlation by Dr. RIG.pptx
Dr. Rabia Inam Gandapore
 
Best Ayurvedic medicine for Gas and Indigestion
Best Ayurvedic medicine for Gas and IndigestionBest Ayurvedic medicine for Gas and Indigestion
Best Ayurvedic medicine for Gas and Indigestion
SwastikAyurveda
 
Non-respiratory Functions of the Lungs.pdf
Non-respiratory Functions of the Lungs.pdfNon-respiratory Functions of the Lungs.pdf
Non-respiratory Functions of the Lungs.pdf
MedicoseAcademics
 
basicmodesofventilation2022-220313203758.pdf
basicmodesofventilation2022-220313203758.pdfbasicmodesofventilation2022-220313203758.pdf
basicmodesofventilation2022-220313203758.pdf
aljamhori teaching hospital
 
SURGICAL ANATOMY OF THE RETROPERITONEUM, ADRENALS, KIDNEYS AND URETERS.pptx
SURGICAL ANATOMY OF THE RETROPERITONEUM, ADRENALS, KIDNEYS AND URETERS.pptxSURGICAL ANATOMY OF THE RETROPERITONEUM, ADRENALS, KIDNEYS AND URETERS.pptx
SURGICAL ANATOMY OF THE RETROPERITONEUM, ADRENALS, KIDNEYS AND URETERS.pptx
Bright Chipili
 
Basavarajeeyam - Ayurvedic heritage book of Andhra pradesh
Basavarajeeyam - Ayurvedic heritage book of Andhra pradeshBasavarajeeyam - Ayurvedic heritage book of Andhra pradesh
Basavarajeeyam - Ayurvedic heritage book of Andhra pradesh
Dr. Madduru Muni Haritha
 
Physiology of Special Chemical Sensation of Taste
Physiology of Special Chemical Sensation of TastePhysiology of Special Chemical Sensation of Taste
Physiology of Special Chemical Sensation of Taste
MedicoseAcademics
 
Hemodialysis: Chapter 3, Dialysis Water Unit - Dr.Gawad
Hemodialysis: Chapter 3, Dialysis Water Unit - Dr.GawadHemodialysis: Chapter 3, Dialysis Water Unit - Dr.Gawad
Hemodialysis: Chapter 3, Dialysis Water Unit - Dr.Gawad
NephroTube - Dr.Gawad
 
Maxilla, Mandible & Hyoid Bone & Clinical Correlations by Dr. RIG.pptx
Maxilla, Mandible & Hyoid Bone & Clinical Correlations by Dr. RIG.pptxMaxilla, Mandible & Hyoid Bone & Clinical Correlations by Dr. RIG.pptx
Maxilla, Mandible & Hyoid Bone & Clinical Correlations by Dr. RIG.pptx
Dr. Rabia Inam Gandapore
 
Pictures of Superficial & Deep Fascia.ppt.pdf
Pictures of Superficial & Deep Fascia.ppt.pdfPictures of Superficial & Deep Fascia.ppt.pdf
Pictures of Superficial & Deep Fascia.ppt.pdf
Dr. Rabia Inam Gandapore
 
NVBDCP.pptx Nation vector borne disease control program
NVBDCP.pptx Nation vector borne disease control programNVBDCP.pptx Nation vector borne disease control program
NVBDCP.pptx Nation vector borne disease control program
Sapna Thakur
 
Local Advanced Lung Cancer: Artificial Intelligence, Synergetics, Complex Sys...
Local Advanced Lung Cancer: Artificial Intelligence, Synergetics, Complex Sys...Local Advanced Lung Cancer: Artificial Intelligence, Synergetics, Complex Sys...
Local Advanced Lung Cancer: Artificial Intelligence, Synergetics, Complex Sys...
Oleg Kshivets
 
263778731218 Abortion Clinic /Pills In Harare ,
263778731218 Abortion Clinic /Pills In Harare ,263778731218 Abortion Clinic /Pills In Harare ,
263778731218 Abortion Clinic /Pills In Harare ,
sisternakatoto
 
The Electrocardiogram - Physiologic Principles
The Electrocardiogram - Physiologic PrinciplesThe Electrocardiogram - Physiologic Principles
The Electrocardiogram - Physiologic Principles
MedicoseAcademics
 

Recently uploaded (20)

Superficial & Deep Fascia of the NECK.pptx
Superficial & Deep Fascia of the NECK.pptxSuperficial & Deep Fascia of the NECK.pptx
Superficial & Deep Fascia of the NECK.pptx
 
Pharynx and Clinical Correlations BY Dr.Rabia Inam Gandapore.pptx
Pharynx and Clinical Correlations BY Dr.Rabia Inam Gandapore.pptxPharynx and Clinical Correlations BY Dr.Rabia Inam Gandapore.pptx
Pharynx and Clinical Correlations BY Dr.Rabia Inam Gandapore.pptx
 
ARTHROLOGY PPT NCISM SYLLABUS AYURVEDA STUDENTS
ARTHROLOGY PPT NCISM SYLLABUS AYURVEDA STUDENTSARTHROLOGY PPT NCISM SYLLABUS AYURVEDA STUDENTS
ARTHROLOGY PPT NCISM SYLLABUS AYURVEDA STUDENTS
 
Ozempic: Preoperative Management of Patients on GLP-1 Receptor Agonists
Ozempic: Preoperative Management of Patients on GLP-1 Receptor Agonists  Ozempic: Preoperative Management of Patients on GLP-1 Receptor Agonists
Ozempic: Preoperative Management of Patients on GLP-1 Receptor Agonists
 
Pharma Pcd Franchise in Jharkhand - Yodley Lifesciences
Pharma Pcd Franchise in Jharkhand - Yodley LifesciencesPharma Pcd Franchise in Jharkhand - Yodley Lifesciences
Pharma Pcd Franchise in Jharkhand - Yodley Lifesciences
 
Flu Vaccine Alert in Bangalore Karnataka
Flu Vaccine Alert in Bangalore KarnatakaFlu Vaccine Alert in Bangalore Karnataka
Flu Vaccine Alert in Bangalore Karnataka
 
Triangles of Neck and Clinical Correlation by Dr. RIG.pptx
Triangles of Neck and Clinical Correlation by Dr. RIG.pptxTriangles of Neck and Clinical Correlation by Dr. RIG.pptx
Triangles of Neck and Clinical Correlation by Dr. RIG.pptx
 
Best Ayurvedic medicine for Gas and Indigestion
Best Ayurvedic medicine for Gas and IndigestionBest Ayurvedic medicine for Gas and Indigestion
Best Ayurvedic medicine for Gas and Indigestion
 
Non-respiratory Functions of the Lungs.pdf
Non-respiratory Functions of the Lungs.pdfNon-respiratory Functions of the Lungs.pdf
Non-respiratory Functions of the Lungs.pdf
 
basicmodesofventilation2022-220313203758.pdf
basicmodesofventilation2022-220313203758.pdfbasicmodesofventilation2022-220313203758.pdf
basicmodesofventilation2022-220313203758.pdf
 
SURGICAL ANATOMY OF THE RETROPERITONEUM, ADRENALS, KIDNEYS AND URETERS.pptx
SURGICAL ANATOMY OF THE RETROPERITONEUM, ADRENALS, KIDNEYS AND URETERS.pptxSURGICAL ANATOMY OF THE RETROPERITONEUM, ADRENALS, KIDNEYS AND URETERS.pptx
SURGICAL ANATOMY OF THE RETROPERITONEUM, ADRENALS, KIDNEYS AND URETERS.pptx
 
Basavarajeeyam - Ayurvedic heritage book of Andhra pradesh
Basavarajeeyam - Ayurvedic heritage book of Andhra pradeshBasavarajeeyam - Ayurvedic heritage book of Andhra pradesh
Basavarajeeyam - Ayurvedic heritage book of Andhra pradesh
 
Physiology of Special Chemical Sensation of Taste
Physiology of Special Chemical Sensation of TastePhysiology of Special Chemical Sensation of Taste
Physiology of Special Chemical Sensation of Taste
 
Hemodialysis: Chapter 3, Dialysis Water Unit - Dr.Gawad
Hemodialysis: Chapter 3, Dialysis Water Unit - Dr.GawadHemodialysis: Chapter 3, Dialysis Water Unit - Dr.Gawad
Hemodialysis: Chapter 3, Dialysis Water Unit - Dr.Gawad
 
Maxilla, Mandible & Hyoid Bone & Clinical Correlations by Dr. RIG.pptx
Maxilla, Mandible & Hyoid Bone & Clinical Correlations by Dr. RIG.pptxMaxilla, Mandible & Hyoid Bone & Clinical Correlations by Dr. RIG.pptx
Maxilla, Mandible & Hyoid Bone & Clinical Correlations by Dr. RIG.pptx
 
Pictures of Superficial & Deep Fascia.ppt.pdf
Pictures of Superficial & Deep Fascia.ppt.pdfPictures of Superficial & Deep Fascia.ppt.pdf
Pictures of Superficial & Deep Fascia.ppt.pdf
 
NVBDCP.pptx Nation vector borne disease control program
NVBDCP.pptx Nation vector borne disease control programNVBDCP.pptx Nation vector borne disease control program
NVBDCP.pptx Nation vector borne disease control program
 
Local Advanced Lung Cancer: Artificial Intelligence, Synergetics, Complex Sys...
Local Advanced Lung Cancer: Artificial Intelligence, Synergetics, Complex Sys...Local Advanced Lung Cancer: Artificial Intelligence, Synergetics, Complex Sys...
Local Advanced Lung Cancer: Artificial Intelligence, Synergetics, Complex Sys...
 
263778731218 Abortion Clinic /Pills In Harare ,
263778731218 Abortion Clinic /Pills In Harare ,263778731218 Abortion Clinic /Pills In Harare ,
263778731218 Abortion Clinic /Pills In Harare ,
 
The Electrocardiogram - Physiologic Principles
The Electrocardiogram - Physiologic PrinciplesThe Electrocardiogram - Physiologic Principles
The Electrocardiogram - Physiologic Principles
 

TEST BANK for Wilkins’ Clinical Assessment in Respiratory Care, 9th Edition by Albert.pdf

  • 1. TO GET ALL CHAPTERS EMAIL ME AT>>>>> donc8246@gmail.com Test bank For Wilkins clinical assessment in respiratory care 8th edition by Huber, Chapters 1 - 21
  • 2. TO GET ALL CHAPTERS EMAIL ME AT>>>>> donc8246@gmail.com Wilkins' Clinical Assessment in Respiratory Care, 7th Edition Contents: Chapter 1. Preparing for the Patient Encounter Chapter 2. The Medical History and the Interview Chapter 3. Cardiopulmonary Symptoms Chapter 4. Vital Signs Chapter 5. Fundamentals of Physical Examination Chapter 6. Neurologic Assessment Chapter 7. Clinical Laboratory Studies Chapter 8. Interpretation of Blood Gases Chapter 9. Pulmonary Function Testing Chapter 10. Chest Imaging Chapter 11. Electrocardiography Chapter 12. Neonatal and Pediatric Assessment Chapter 13. Older Patient Assessment Chapter 14. Monitoring in Critical Care Chapter 15. Vascular Pressure Monitoring Chapter 16. Cardiac Output Measurement Chapter 17. Bronchoscopy Chapter 18. Nutritional Assessment Chapter 19. Sleep and Breathing Assessment Chapter 20. Home Care Patient Assessment Chapter 21. Documentation
  • 3. TO GET ALL CHAPTERS EMAIL ME AT>>>>> donc8246@gmail.com Chapter 1: Preparing for the Patient Encounter Test Bank MULTIPLE CHOICE 1. Which of the following activities is not part of the role of respiratory therapists (RTs) in patient assessment? a. Assist the physician with diagnostic reasoning skills. b. Help the physician select appropriate pulmonary function tests. c. Interpret arterial blood gas values and suggest mechanical ventilation changes. d. Document the patient diagnosis in the patient’s chart. ANSWER: D RTs are not qualified to make an official diagnosis. This is the role of the attending physician. REF: Table 1-1, pg. 4 OBJ: 9 2. In which of the following stages of patient–clinician interaction is the review of physician orders carried out? a. Treatment stage b. Introductory stage c. Preinteraction stage d. Initial assessment stage ANSWER: C Physician orders should be reviewed in the patient’s chart before the physician sees the patient. REF: Table 1-1, pg. 4 OBJ: 9 3. In which stage of patient–clinician interaction is the patient identification bracelet checked? a. Introductory stage b. Preinteraction stage c. Initial assessment stage d. Treatment stage ANSWER: A The patient ID bracelet must be checked before moving forward with assessment and treatment. REF: Table 1-1, pg. 4 OBJ: 9 4. What should be done just before the patient’s ID bracelet is checked? a. Check the patient’s SpO2. b. Ask the patient for permission. c. Check the chart for vital signs. d. Listen to breath sounds. ANSWER: B It is considered polite to ask the patient for permission before touching and reading his or her ID bracelet.
  • 4. TO GET ALL CHAPTERS EMAIL ME AT>>>>> donc8246@gmail.com REF: pg. 3 OBJ: 3 | 5 5. What is the goal of the introductory phase? a. Assess the patient’s apparent age. b. Identify the patient’s family history. c. Determine the patient’s diagnosis. d. Establish a rapport with the patient. ANSWER: D The introductory phase is all about getting to know the patient and establishing a rapport with him or her. REF: Table 1-1, pg. 4 OBJ: 3 6. Which of the following behaviors is not consistent with resistive behavior of a patient? a. Crossed arms b. Minimal eye contact c. Brief answers to questions d. Asking the purpose of the treatment ANSWER: D If a patient asks about the purpose of the treatment you are about to give, this generally indicates that he or she is not upset. REF: Table 1-1, pg. 4 OBJ: 3 7. What is the main purpose of the initial assessment stage? a. To identify any allergies to medications b. To document the patient’s smoking history c. To personally get to know the patient better d. To verify that the prescribed treatment is still needed and appropriate ANSWER: D When you first see the patient, you are encouraged to perform a brief assessment to make sure the treatment order by the physician is still appropriate. The patient’s status may have changed abruptly recently. REF: Table 1-1, pg. 4 OBJ: 3 8. What is the appropriate distance for the social space from the patient? a. 3 to 5 feet b. 4 to 12 feet c. 6 to 18 feet d. 8 to 20 feet ANSWER: B The social space is 4 to 12 feet. REF: pg. 5 OBJ: 5 9. What is the appropriate distance for the personal space?
  • 5. TO GET ALL CHAPTERS EMAIL ME AT>>>>> donc8246@gmail.com a. 0 to 18 inches b. 18 inches to 4 feet c. 4 to 12 feet d. 6 to 15 feet ANSWER: B The personal space is about 2 to 4 feet from the patient. REF: pg. 5 OBJ: 5 10. Which of the following activities is best performed in the personal space? a. The interview b. The introduction c. The physical examination d. Listening for breath sounds ANSWER: A The interview is best performed with you sitting about 2 to 4 feet from the patient. If you sit farther away, the patient will have to answer your questions in a louder voice, and because some of the information may be private, this would diminish communication. REF: Table 1-1, pg. 4 OBJ: 5 11. What type of behavior is least appropriate in the patient’s intimate space? a. Eye contact b. Pulse check c. Auscultation d. Simple commands ANSWER: A Eye contact is inappropriate in the intimate space and will make the patient very uncomfortable. REF: pg. 3 OBJ: 5 12. You are riding in an elevator at the hospital where you are employed as an RT. The elevator is full, but standing next to you is Joe, the RT who is scheduled to relieve you. He turns to you and asks, “How is Mr. Copper doing?” Earlier in the day, Mr. Copper had a cardiac arrest, and he is now being mechanically ventilated. How should you respond to Joe? a. “He took a turn for the worse.” b. “He is fine.” c. “Let’s talk later in the report room.” d. “He is on a ventilator and will keep you very busy.” ANSWER: C The patient’s right to privacy prevents care providers from discussing a patient’s clinical status in public places. All answers other than “c” are unethical; giving such answers could cause an RT to be in legal trouble and get fired. REF: Table 1-2, pgs. 4-5 OBJ: 6 13. In 1996, Congress passed the HIPAA. What does the letter “P” stand for?
  • 6. TO GET ALL CHAPTERS EMAIL ME AT>>>>> donc8246@gmail.com a. Patient b. Payment c. Portability d. Personal ANSWER: C HIPAA stands for Health Insurance Portability and Accountability Act. REF: pg. 4 OBJ: 6 14. Which of the following techniques for expressing genuine concern is the most difficult to use appropriately? a. Touch b. Posture c. Eye contact d. Proper introductions ANSWER: A Touch is most difficult to use properly because gender and cultural differences often become an issue. REF: Table 1-1, pgs. 3-4 OBJ: 5 | 7 15. Which of the following techniques is not associated with the demonstration of active listening? a. Good eye contact b. Taking notes while a patient is talking c. Asking for clarification d. Use of touch ANSWER: D Use of touch helps with demonstrating empathy but has little to do with active listening. REF: pg. 2 OBJ: 2 16. Two respiratory care students are taking their lunch break and want to compare notes about patients they have seen during the morning. Which of the following locations would be considered a violation of HIPAA standards? a. The unit nursing station in front of the unit clerk’s desk b. A table in the cafeteria with no one within hearing distance c. The respiratory department report room d. The intensive care unit (ICU) staff break room ANSWER: A Patient Health Information (PHI) should be discussed only in nonpublic areas of the hospital. The space in front of the unit clerk’s desk is likely to be occupied with members of the public asking for information. REF: Table 1-2, pgs. 4-5 OBJ: 6
  • 7. TO GET ALL CHAPTERS EMAIL ME AT>>>>> donc8246@gmail.com 17. A 20-year-old respiratory care student enters the room of a 65-year-old female patient, saying, “Hi, Linda! I am Joe from Respiratory Care.” He immediately approaches her, looks her in the eye, and places his stethoscope on her chest. a. Joe’s approach to this patient is appropriate. b. Joe has inappropriately entered the patient’s social space. c. Joe has inappropriately entered the patient’s personal space. d. Joe has inappropriately entered the patient’s intimate space. ANSWER: D Joe has established no rapport with this patient, has touched her without asking permission, and looks her in the eye while examining her. REF: pg. 4 OBJ: 7 18. A respiratory care student returns from a clinical experience , excited that she has had the opportunity to perform cardiopulmonary resuscitation (CPR) for the first time. She immediately goes to her Facebook page and describes her day. Which of the following entries would be a violation of HIPAA standards? a. “At clinical today got to do CPR on a patient on the 6th floor of Mercy Hospital. Patient survived! What a rush!!” b. “Got to do CPR for the first time today. Patient survived!! What a rush!” c. “Got to do CPR for the first time in clinical today! What a rush!!” d. All of the above ANSWER: D Patient Health Information (PHI) must not be shared in a public location. Facebook is considered a public forum. Although the student did not give specific identifiers in answers a, b, or c, there was enough information that someone familiar with either the patient or the student could possibly have deduced the identity of the patient. REF: Table 1-2, pgs. 4-5 OBJ: 6 19. Which of the following would be the most appropriate way for respiratory care student Amy Long to initially approach a 58-year-old female patient, Mrs. Nora Jones? a. “Hello, Mrs. Jones. I am Amy from respiratory care, and with your permission I would like to assess you for your treatment.” (Amy stands 5 feet from the patient and makes direct eye contact.) b. Hey there, Nora! Isn’t this a great day!?! I’m Amy and I need to listen to you.” (Amy holds out her stethoscope in front of her and approaches the patient to within 1.5 feet.) c. “Hi, I’m Amy, here to give you your treatment.” (Amy makes no eye contact and looks around the room for a nebulizer.) d. Hi, Mrs. Jones. I’m here for your treatment.” (Amy makes direct eye contact.) ANSWER: A The initial contact with a patient should be from the patient’s social space (4 to 12 feet). Patients should be addressed by their last name. When first speaking to a patient, the therapist should make direct eye contact, but he or she should not use direct eye contact when in the patient’s intimate space. REF: Table 1-1, pgs. 3-4 OBJ: 2 | 7
  • 8. TO GET ALL CHAPTERS EMAIL ME AT>>>>> donc8246@gmail.com 20. The umbrella term patient-centered care includes all of the following elements except: a. Individualized care. b. Assistance with financial and insurance issues. c. Patient involvement. d. Provider collaboration. ANSWER: B Patient-centered care involves individualized care, patient involvement, and provider collaboration. REF: pg. 2 OBJ: 1 21. The golden rule of bedside care can be summarized as: a. Patients should be cared for primarily at the bedside. b. All patient services (e.g., x-ray, nursing care, respiratory care) should be delivered to the patient at the bedside whenever possible. c. As a caregiver, at all times treat a patient as you would hope to be treated if you were the patient. d. Make sure that all safety equipment is in place at the bedside for maximum patient protection from hazards such as falls. ANSWER: C The golden rule is that as a caregiver, you treat patients the way you wish to be treated. REF: pg. 2 OBJ: 1 22. In interacting with patients, behaviors such as body movements, touch and eye movements, and facial expressions would be examples of: a. Nonverbal communication. b. Expressions of caregiver interest in patient welfare. c. Mechanisms to put patients at ease. d. None of the above. ANSWER: A These are mechanisms of nonverbal communication that help to put patients at ease and can be used to communicate caregiver concern to patients. REF: pg. 2 OBJ: 2 23. In determining the course of treatment for a 20-year-old patient hospitalized for exacerbation of cystic fibrosis, the most effective course of action would be: a. Formulating a treatment plan based on the therapist’s knowledge of the disease and its treatment and then presenting it to the patient. b. Formulating a treatment plan with the physician and nurse and then presenting it to the patient. c. Interviewing the patient and strictly following the patient’s preferences with regard to treatment. d. Interviewing the patient to determine his or her preferences for treatment, formulating a treatment plan in collaboration with the nurse and physician based on both patient preferences and the team’s knowledge of the disease and its treatment,
  • 9. TO GET ALL CHAPTERS EMAIL ME AT>>>>> donc8246@gmail.com and presenting it to the patient. ANSWER: D Patient-centered care must be highly collaborative, with input from both the patient and caregivers. REF: pg. 7 OBJ: 4 | 8 24. While interviewing a patient in a room with another patient and that patient’s family present, the appropriate course of action for an RT would be to: a. Introduce himself or herself to the patient from a distance of about 5 to 7 feet and proceed with the interview. b. Introduce himself or herself to the patient from a distance of about 5 to 7 feet, move to within 2 to 3 feet of the patient, and proceed with the interview. c. Introduce himself or herself to the patient from a distance of about 5 to 7 feet, move to within 2 to 3 feet of the patient, draw the privacy curtain around the bed, and proceed with the interview. d. Introduce himself or herself to the patient from a distance of about 5 to 7 feet, draw the privacy curtain around the bed, sit on the bed about 1 foot from the patient, and proceed with the interview. ANSWER: C Normally, interviews are carried out in the personal space (within 18 inches to 4 feet of the patient), not the intimate space (from 1 to 18 inches from the patient). The privacy curtain should be in place because others are in the room. REF: pg. 3 OBJ: 3 25. A male therapist is discussing a treatment plan with a female patient who is sitting up in bed, dressed in a hospital gown and wearing a full head covering with only her face showing. Her husband is in the room, and from previous encounters it is clear that she defers to him. The most effective way to present this treatment plan would be for the therapist to: a. Present the patient with a written summary of the plan and ask her to look it over. b. Ask the woman’s husband to step out of the room while the plan is being discussed with the patient. c. Explain the plan to the patient, maintaining eye contact with her at all times and encouraging her to ask any questions she might have. d. Explain the plan to the patient and her husband, and encourage both to ask any questions they might have. ANSWER: D From the woman’s dress and previous behavior, it is likely that she is Muslim, with traditional values and customs. Therefore, both she and her husband will find it inappropriate, if not offensive, that he not be included in discussions of treatment. Cultural values must be taken into account if truly effective patient treatment is to occur. REF: pg. 5 OBJ: 4 | 7
  • 10. TO GET ALL CHAPTERS EMAIL ME AT>>>>> donc8246@gmail.com 26. The therapist enters the room of a 6-year-old victim of an automobile accident who is unconscious and receiving ventilation therapy. The therapist assesses the patient, gives a treatment, and suctions the patient. The patient’s mother then asks in a worried voice, “Is he going to be all right?” The appropriate response for the therapist would be: a. “I just looked at the CT scan of his brain, and I believe the swelling is going down. He should recover within the next couple of weeks.” b. “I’m unable to give you any information about your child’s condition.” c. “I am just the respiratory therapist, and I really do not know anything.” d. “I’m sorry, but our policy is that only the doctor can give you information about your child’s prognosis. Let me step out and find out when the doctor will be back in the unit.” ANSWER: D Response “a” is inappropriate because it is not within the scope of practice of an RT to render medical opinions about a patient’s condition to the family. Answer “b,” while correct, is unnecessarily abrupt. Answer “c” is both abrupt and is probably not true. Answer “d” meets HIPAA and hospital policy requirements while also providing good patient care and customer service. REF: pgs. 4-5 OBJ: 6 27. In order to deliver effective patient education for use of a particular treatment, the first step should be to: a. Describe to the patient the equipment that will be used for the treatments. b. Describe to the patient the medications that will be used for the treatments. c. Assess the patient’s learning needs by identifying learning barriers, determining the way the patient best learns, and evaluating the patient’s readiness to learn. d. Describe to the patient the schedule for the treatments to be given. ANSWER: C Although the patient will eventually have to learn about equipment, medications, and schedules, this learning will not occur effectively until the patient’s learning needs are determined. REF: pgs. 6-7 OBJ: 9 28. One effective teaching tool is the teach-back method. This is: a. A technique where the teacher explains the procedure to the learner and has the learner repeat the information in his or her own words. b. A technique where the teacher explains the procedure to the learner and then has the learner explain it to family members after the teacher has left the room. c. A technique where the teacher gives the equipment to the learner and has the learner experiment with it until he or she can use it correctly. d. None of the above. ANSWER: A The teach-back method has the learner hear the explanation and then give a “return demonstration” to the teacher to be sure that the learner has the correct information. REF: pg. 7 OBJ: 9
  • 11. TO GET ALL CHAPTERS EMAIL ME AT>>>>> donc8246@gmail.com 29. Prior to discharge, patients should receive a written action plan that establishes treatment goals and self-care activities. The acronym SMART is helpful in establishing the action plan. The “M” in SMART stands for: a. Meaningful (the goal pertains to the action plan). b. Mastering (the goal). c. Modular (the goal is divided into sections). d. Measurable (the outcome should be measurable). ANSWER: D In order for a treatment to be effective, its outcome must be measurable. REF: pg. 7 OBJ: 10 30. Failure of care providers to collaborate with one another potentially results in: a. Patient safety placed at risk. b. Duplication of effort by different caregivers. c. Delivery of less-than-optimum care. d. All of the above. ANSWER: D In addition to “a,” “b,” and “c,” lack of collaboration also may result in an increased length of stay and wasted healthcare resources. REF: pgs. 9-11 OBJ: 13 | 14 | 15 31. An RT enters a room for a patient’s second treatment of the day and notes that the patient is dyspneic and tachycardic and has rales that can be heard throughout all lung fields but are especially prominent in the bases. A treatment is given with little positive change. The therapist should: a. Note the treatment and its results in the patient chart. b. Note the treatment and its results in the patient chart and tell the unit clerk to have the nurse see the patient soon. c. Find the patient’s nurse and together determine what is needed for the patient (e.g., different drug therapy, call physician, call rapid response team). Once the patient’s condition has been adequately addressed, the therapist’s actions should be documented in the chart. d. Tell the patient you will check back with him in an hour to see how he is doing. ANSWER: C It is critical that results of an assessment and treatment be communicated to other members of the health team, either in the chart or verbally if the situation demands immediate action. Chart documentation should include findings of the assessment, results of any treatments delivered, and actions taken to resolve the situation. REF: pg. 10 OBJ: 15 32. Good communication is especially critical when “handing off” a patient to another caregiver at the end of the shift. To ensure that adequate information is transmitted, the SBAR format is useful. This acronym stands for situation, background, assessment, and . a. Recommendation. b. Results.
  • 12. TO GET ALL CHAPTERS EMAIL ME AT>>>>> donc8246@gmail.com c. References. d. Repeat (information) ANSWER: A The fourth letter in SBAR stands for recommendation. REF: pg. 10 OBJ: 14 33. According to the American Association for Respiratory Care (AARC), patient discharge plans should include which of the following? a. Transportation for the patient when the patient goes home b. Ascertaining that once the patient has been discharged, the patient has adequate financial resources to pay rent and utilities c. Notification of the patient’s family and/or other caregivers of the patient’s imminent discharge d. Methods for the ongoing assessment of outcomes ANSWER: D Discharge planning should focus on providing a continuum of care with transition from the hospital to the alternate site. REF: pg. 11 OBJ: 14 34. All of the following represent good hygiene practices except: a. Gloves should be worn when touching a patient only when a caregiver is likely to come into contact with secretions or infectious materials. b. Hands should be washed when first encountering a patient, after leaving a patient, and before and after any sterile or clean procedure is performed on a patient. c. All infectious waste should be disposed of in proper waste containers. d. Place all needles and blades in “sharps” containers when you are finished with them. ANSWER: A Gloves should be worn whenever a patient is touched. REF: pg. 9, Box 1-4 OBJ: 12 35. The main objective of the I Speak Up initiative from the National Institute of Health (NIH) is: a. Be sure that all billing for patient medical expenses occurs correctly. b. Let caregivers know that the family wants to be informed at all times. c. Let the hospital know when ancillary services such as food quality and parking are inadequate. d. Help ensure that a patient’s care is as safe and effective as possible. ANSWER: D The I Speak Up initiative is a comprehensive program focused on making sure that patient care is as safe and effective as possible. It especially emphasizes the importance of active patient and family involvement in all aspects of patient care, including enhanced patient safety and reduction in medical errors. REF: pg. 8, Box 1-3 OBJ: 11
  • 13. TO GET ALL CHAPTERS EMAIL ME AT>>>>> donc8246@gmail.com Chapter 2: The Medical History and the Interview Test Bank MULTIPLE CHOICE 1. Communication between two people can occur only if: a. The speaker speaks clearly. b. The receiver understands the message. c. Each person is willing to listen to the other. d. All of the above. ANSWER: D Communication occurs only if both parties speak clearly and listen carefully. REF: pg. 16 OBJ: 1 2. Communication between individuals is affected by all of the following factors except: a. The time of day. b. The cultural heritage. c. The religious beliefs. d. The level of education. ANSWER: A The time of day is not likely to affect communication in a significant way. REF: pg. 16 OBJ: 2 3. When one is conducting an interview with a patient, which of the following points is most important in facilitating an effective interaction with the patient? a. Recognizing the nonverbal signals that the patient is sending b. Your ability to project a sense of undivided interest in the patient c. Introducing yourself appropriately at the beginning of the interview d. Answering all of the patient’s questions completely without using jargon ANSWER: B Patients can sense when we are distracted and will not communicate well in such a situation. Glancing out the window or at the television will tell the patient that you are distracted and are not that interested in what he or she has to say. REF: pgs. 16-17 OBJ: 3 4. Which of the following types of questions are preferred for all interactions with a patient? a. Direct questions b. Neutral questions c. Indirect questions d. Open-ended questions ANSWER: B Neutral questions encourage the patient to respond with sentences and honest answers. REF: pg. 17 OBJ: 3
  • 14. TO GET ALL CHAPTERS EMAIL ME AT>>>>> donc8246@gmail.com 5. If a patient is unable to provide an accurate history, the respiratory therapist (RT) should: a. Ask a family member or friend to supply the information. b. Ask shorter and more direct questions to determine the cause of the problem. c. Proceed to treat the patient’s symptoms based on information already obtained. d. Refuse to treat the patient because effective therapy cannot be initiated without a complete history. ANSWER: A In many situations, the patient is unable to answer questions. Family members are often the next best source for important information. REF: pg. 18 OBJ: 4 6. Which of the following should the RT keep in mind when obtaining a pulmonary history? a. Assessment usually is limited to the respiratory system. b. Evaluation of the patient’s entire health status is essential. c. Signs and symptoms of pulmonary disease will rarely be seen outside the cardiopulmonary system. d. Patients with long-standing chronic disease can give a detailed account of how their lives have changed and of the signs and symptoms that the disease has caused. ANSWER: B Pulmonary problems often cause health issues in other body systems, and health problems in other systems often cause pulmonary problems. REF: pg. 18 OBJ: 2 | 6 7. Obtaining background information during an interview is very important because it allows the interviewer to: a. Learn the impact of culture, relationships, and finances on his or her health. b. Predict whether the patient is willing to cooperate in the treatment of his or her disease. c. Develop a basic understanding of the patient’s experience with his or her disease. d. Achieve all of the above. ANSWER: D Background information is often overlooked but is important for assessing the impact of chronic illness on the patient and for identifying how the patient is coping with the illness. It also tells the interviewer whether the patient is able to cooperate with the treatment plan. REF: pg. 18 OBJ: 2 | 6 8. Screening information is: a. Obtained at health fairs to determine whether the person should see a physician. b. Designed to uncover problem areas that the patient forgot to mention or omitted. c. Given to the interviewer by the patient at the very beginning of the interview process. d. Obtained by the triage nurse in the emergency department to determine how life- threatening the patient’s symptoms are. ANSWER: B
  • 15. TO GET ALL CHAPTERS EMAIL ME AT>>>>> donc8246@gmail.com Screening information is designed to identify important facts that the patient may have overlooked in the interview. REF: pgs. 18-19 OBJ: 2 9. The review of systems is very important because it provides the interviewer with: a. Information grouped by major organs and physiologic systems. b. Additional subjective information about the patient’s problem. c. A combination of subjective and objective data in a narrative form. d. Information relevant to the patient’s problem that may have been overlooked. ANSWER: D The review of systems helps identify key information that may have been overlooked in the interview. REF: pg. 19 OBJ: 2 10. A pertinent negative is defined as: a. Any negative response made by the patient during the interview. b. Refusal by the patient to answer questions about a certain topic. c. Any negative response by the patient to an important question about possible symptoms. d. A negative response by a patient to a therapist who asks whether the patient needs a breathing treatment. ANSWER: C If the patient appears to have pneumonia but denies having a cough, the patient’s negative response to the question about coughing would be a pertinent negative. REF: pg. 19 OBJ: 3 11. A pertinent positive is defined as: a. Any positive response made by the patient during the interview. b. A direct question asked during the interview that elicits a positive response. c. An affirmative response by the patient when asked whether he or she needs a breathing treatment. d. An affirmative response to an important interview question about the patient’s symptoms. ANSWER: D A positive response from the patient regarding a symptom associated with the possible diagnosis would be a pertinent positive. REF: pg. 19 OBJ: 3 12. The main purpose of the chief complaint is to: a. Give a brief explanation about why the patient sought health care. b. Direct the interviewer to the organ system in which the problem is located. c. Present a diagnosis that is based on information obtained during the interview. d. List symptoms in order from most severe to least severe according to organ system involvement.
  • 16. TO GET ALL CHAPTERS EMAIL ME AT>>>>> donc8246@gmail.com ANSWER: A The chief complaint explains why the patient sought medical help. REF: pg. 19 OBJ: 2 | 3 13. Which of the following cardiopulmonary conditions would not be found in the chief complaint list? a. Asthma b. Wheezing c. Chest pain d. Hemoptysis ANSWER: A Asthma is a diagnosis and not a complaint. REF: pg. 19 | pg. 21 OBJ: 2 14. When a patient is interviewed so the chief complaint can be determined, the best questions that can be asked to elicit this information are: a. Direct questions. b. Neutral questions. c. Open-ended questions. d. A combination of the above. ANSWER: D Use of a variety of types of questions usually yields the best interview results. REF: pg. 17 | pg. 19 OBJ: 2 15. Which of the following would not be found in the past medical history? a. Injuries and accidents b. Surgeries and hospitalizations c. Associated symptoms and aggravating factors d. Over-the-counter medications, vitamins, and “home remedies” ANSWER: C Associated symptoms and aggravating factors would be found in the history of present illness. REF: pg. 22 OBJ: 2 16. Which of the following formulas should be used to calculate the pack-year history of cigarette consumption? a. Packs per day times Years smoked b. Packs per day minus Years smoked c. Packs per day plus Years smoked d. Packs per day plus Years smoked ANSWER: A Pack-years is the standard way to document a patient’s smoking history. This is determined by multiplying the number of packs smoked times the number of years smoked. REF: pg. 24 OBJ: 2
  • 17. TO GET ALL CHAPTERS EMAIL ME AT>>>>> donc8246@gmail.com 17. One disadvantage of using the pack-year method for calculating cigarette consumption is that: a. The method is not used widely throughout the United States. b. Advanced mathematical calculations are required to obtain the figure. c. The method does not reveal how many packs per day were smoked over how many years. d. The values are more meaningful to RTs and physicians than to other healthcare providers. ANSWER: C Although pack-years is an industry standard for documenting quantity of tobacco consumption, this value does not provide the details. REF: pg. 24 OBJ: 2 18. Which of the following is not a reason for obtaining a family history? a. To assess the current health status of the extended family b. To learn about the health status of the patient’s blood relatives c. To determine whether the patient is adopted d. To identify the presence in the family of diseases with a hereditary tendency ANSWER: C The reason for obtaining a family history is to identify blood relatives who may have a disease that the patient has inherited. REF: pg. 25 OBJ: 2 19. Which of the following diseases would not be recorded in the family history as a hereditary disorder? a. Asthma b. Pneumonia c. Cystic fibrosis d. Alpha1-antitrypsin deficiency ANSWER: B Pneumonia is not hereditary. REF: pg. 25 OBJ: 2 20. Modern-day office workers may be exposed to which of the following occupational and environmental diseases? a. Bagassosis b. Monday fever c. Pneumoconiosis d. Sick building syndrome ANSWER: D Sick building syndrome occurs when employees are exposed to low doses of toxic gases in a building that is airtight and has poor ventilation. REF: Table 2-1, pg. 27 OBJ: 2
  • 18. TO GET ALL CHAPTERS EMAIL ME AT>>>>> donc8246@gmail.com 21. Which of the following symptoms is often seen in patients with tight building syndrome? a. Headache b. Stuffy nose c. Cough d. All of the above ANSWER: D Tight building syndrome can cause a variety of symptoms. REF: Table 2-1, pg. 27 OBJ: 2 22. What pulmonary disorder is associated with visiting or living in Ohio, Maryland, and the central Mississippi Valley? a. Blastomycosis b. Histoplasmosis c. Coccidioidomycosis d. Silicosis ANSWER: B Histoplasmosis occurs only in the midwestern regions of the country. REF: Table 2-1, pg. 28 OBJ: 2 23. Who writes the initial admission note? a. The physician b. The head nurse c. The head RT d. Any of the above ANSWER: A Only the physician writes the initial admission note, which describes why the patient is being admitted. REF: pg. 28 OBJ: 2 | 6 24. Who writes the progress notes each day? a. The physician b. The physical therapist c. The nurse d. Any of the above ANSWER: D Any clinician who provides treatment to the patient can write a progress note in the chart. REF: pg. 28 OBJ: 2 | 6 25. A good interview should contain all of the following elements except: a. The interviewer should dress and act professionally. b. The interviewer should project a sense of undivided interest. c. The interviewer should use a formal speaking style. d. The interviewer should respect the patient’s beliefs and attitudes. ANSWER: C
  • 19. TO GET ALL CHAPTERS EMAIL ME AT>>>>> donc8246@gmail.com The interviewer should use an informal, relaxed, conversational style. REF: pg. 17 OBJ: 2 | 3 26. In the physical examination, objective data gathered are referred to as: a. Measurements. b. Symptoms. c. Variables. d. Signs. ANSWER: D Objective data are referred to as “signs” in the physical examination. These are data that can be perceived by the examiner, either by measurements or observations. REF: pg. 19 OBJ: 5 27. In the physical examination, subjective data gathered are referred to as: a. Measurements. b. Symptoms. c. Variables. d. Signs. ANSWER: B Subjective data in the physical examination are referred to as symptoms. These are data that can only be perceived by the patient. A good interviewer, however, is able to ask questions that prompt the patient to give complete and accurate descriptions of his or her symptoms. REF: pg. 19 OBJ: 5 28. An RT examining a patient auscultates wheezes in the right lower lobe (RLL). This would be an example of: a. Objective data (a sign). b. Objective data (a symptom). c. Subjective data (a sign). d. Subjective data (a symptom). ANSWER: A Because wheezes are perceived by the examiner, this would be an example of a sign. REF: pg. 19 OBJ: 5 29. A patient being interviewed says that he wakes up nearly every night feeling very hot and with his body bathed in sweat. This fact, along with his hemoptysis, is a very strong indication that he may have tuberculosis. The night sweats would be an example of: a. Objective data (a sign). b. Objective data (a symptom). c. Subjective data (a sign). d. Subjective data (a symptom). ANSWER: D Because the night sweats are perceived only by the patient and then described to the examiner, they would be an example of subjective data (a symptom).
  • 20. TO GET ALL CHAPTERS EMAIL ME AT>>>>> donc8246@gmail.com REF: pg. 19 OBJ: 5 30. A “constitutional” symptom would include all of the following except: a. Chills and fever. b. Anorexia and/or weight loss. c. Fatigue. d. Wheezing. ANSWER: D Constitutional symptoms generally involve the whole body and are not specific to individual systems. Wheezing is definitely specific to the respiratory system. REF: pg. 21 OBJ: 6 31. All of the following questions would be effective in eliciting additional information about a patient’s location and level of pain except for question . a. Earlier, you briefly mentioned some pain around your lower ribs. Could you tell me more about it? b. Earlier, you briefly mentioned some pain around your lower ribs. Could you point to exactly where it is? c. Earlier, you briefly mentioned some pain around your lower ribs. What do you think is causing it? d. Earlier, you briefly mentioned some pain around your lower ribs. Could you rate this pain for me on a scale of 1 to 10, with 1 meaning no pain and 10 meaning the worst pain possible? ANSWER: C A good interviewer is looking for a precise description of a symptom. It is unlikely that the patient has any idea what is causing the pain. REF: pg. 21 OBJ: 2 32. A history of a patient’s tobacco use is important for all of the following reasons except: a. There is a strong relationship between smoking and chronic obstructive pulmonary disease (COPD). b. There is a strong relationship between smoking and the use of illicit drugs such as marijuana, cocaine, and heroin. c. There is a strong relationship between smoking and cardiovascular disease. d. There is a strong relationship between smoking and lung cancer. ANSWER: B There is no published evidence that shows that smoking increases the likelihood that a person will use illicit drugs. There is ample evidence of the relationship between smoking and COPD, cardiovascular disease, and lung cancer. REF: pg. 22 OBJ: 2 | 3 | 6 33. Monday fever is: a. The term describing workers who habitually do not want to return to work after the weekend. b. The tendency of workers to have poor levels of concentration in the workplace after
  • 21. TO GET ALL CHAPTERS EMAIL ME AT>>>>> donc8246@gmail.com the weekend. c. An example of a hypersensitivity reaction to a toxic inhaled substance that is worst after the initial exposure but becomes progressively less pronounced as the exposure continues. d. None of the above. ANSWER: C First exposure to a toxic inhaled substance may cause an immediate reaction (as on Monday following a weekend), which gradually becomes less severe with continued exposure (i.e., exposure throughout the rest of the week). REF: pg. 27 OBJ: 6 34. In which section of the patient record should the following entry appear? “Day 4 of hospitalization. Patient febrile (39o C). Rales in both lung bases; sputum moderate, thick, green, tinged with blood. Plan: Continue ordered antibiotic therapy. Bronchoscopy tomorrow in AM.” a. Admission note b. Physician orders c. Progress notes d. Discharge plan ANSWER: C The physician should see the hospitalized patient at least once daily to identify the patient’s general condition, progress, and response to treatment. These findings are summarized in the progress notes. REF: pg. 28 OBJ: 6 | 7 35. The RT has been called STAT to the bedside of a patient who is in extreme respiratory distress in the intensive care unit (ICU). Shortly after arriving, the patient’s cardiac monitor shows a pattern of ventricular fibrillation, and shortly thereafter the patient ceases breathing. The nurse states that the patient has a do not resuscitate (DNR) order. The RT should take which of the following actions? a. Do nothing, because a DNR order means that no resuscitative measures should be attempted. b. Call a full code and institute full resuscitative measures because the therapist knows that the physician who wrote the DNR order is not working in the unit this week. c. Call a code so that cardiac medications can be given to correct the ventricular fibrillation; the DNR order means that the patient cannot be intubated. d. Place a non–rebreather mask on the patient for “comfort measures.” ANSWER: A A DNR order signed by a physician means that no resuscitative measures should be undertaken. REF: pg. 28 OBJ: 8
  • 22. TO GET ALL CHAPTERS EMAIL ME AT>>>>> donc8246@gmail.com Chapter3:CardiopulmonarySymptoms Test Bank MULTIPLE CHOICE 1. Which of the following sequences of events best describes the cough mechanism? a. Inspiration, closure of glottis, forceful opening of glottis b. Opening of glottis, relaxation of diaphragm, closure of glottis c. Inspiration, forceful opening of glottis, contraction of diaphragm d. Opening of glottis, contraction of diaphragm, explosive release of trapped intrathoracic air ANSWER: A The cough mechanism can be divided into three phases: 1. Inspiratory phase: Reflex opening of the glottis and contraction of the diaphragm and the thoracic and abdominal muscles cause a deep inspiration with a concomitant increase in lung volume accompanied by an increase in the caliber and length of the bronchi. 2. Compression phase: Closure of the glottis and relaxation of the diaphragm while the expiratory muscles contract against the closed glottis can generate very high intrathoracic pressures and narrowing of the trachea and bronchi. 3. Expiratory phase: Opening of the glottis and explosive release of trapped intrathoracic air occur, along with vibration of the vocal cords and of the mucosal lining of the posterior laryngeal wall, which shakes secretions loose from the larynx and moves undesired material out of the respiratory tract. REF: pg. 33 OBJ: 1 2. Which of the following mechanisms does not explain why patients with chronic obstructive pulmonary disease (COPD) have a poor cough? a. Weak muscles of breathing b. Increased elastic recoil c. Airway obstruction d. Glottis function failure ANSWER: B The effectiveness of a cough is reduced when one or more of the following conditions exist: (1) weakness of the inspiratory or expiratory muscles; (2) inability of the glottis to open or close correctly; (3) obstruction, collapsibility, or alteration in shape or contours of the airways; (4) decrease in lung recoil, as occurs with emphysema; and (5) abnormal quantity or quality of mucus production (e.g., thick sputum). REF: pg. 33 OBJ: 1 3. Chronic productive cough is caused most commonly by which of the following clinical conditions? a. Postnasal drip b. Viral infection c. Upper airway infection d. Bronchopulmonary disease
  • 23. TO GET ALL CHAPTERS EMAIL ME AT>>>>> donc8246@gmail.com ANSWER: A Chronic persistent cough is caused most commonly by postnasal drip syndrome, followed by acute asthma, acute exacerbation of COPD, allergic rhinitis, gastroesophageal reflux disease (GERD), chronic bronchitis, bronchiectasis, and other conditions such as left heart failure, bronchogenic cancer, and sarcoidosis. REF: pg. 34 OBJ: 1 4. Which of the following is not a complication of forceful coughing? a. Rib fracture b. Pneumothorax c. Pleural effusion d. Torn chest muscle ANSWER: C The vigorous muscular activity and high intrathoracic pressures created by forceful coughing may produce a number of complications, such as torn chest muscles, rib fractures, disruption of surgical wounds, pneumothorax or pneumomediastinum, syncope (fainting), arrhythmia, esophageal rupture, and urinary incontinence. REF: pg. 34 OBJ: 1 5. Which of the following conditions is not associated with a characteristic “hacking” cough? a. Smoking b. Viral infection c. Nervous habit d. Lung cancer ANSWER: D Hacking (frequent brief periods of coughing or clearing the throat) may be dry and may be the result of smoking, a viral infection, a nervous habit, or difficult-to-move secretions, as occur with postnasal drip. REF: pg. 34 OBJ: 1 6. Which of the following is the best definition of sputum? a. Secretions b. Thick tenacious secretions c. Secretions from the lungs and lower airways d. Secretions from the nose, mouth, and tracheobronchial tree ANSWER: D Sputum is the substance expelled from the tracheobronchial tree, pharynx, mouth, sinuses, and nose by coughing or clearing the throat. REF: pg. 35 OBJ: 1 7. What term is used to describe secretions strictly from the lungs and lower airways? a. Phlegm b. Sputum c. Mucus
  • 24. TO GET ALL CHAPTERS EMAIL ME AT>>>>> donc8246@gmail.com d. Empyema ANSWER: A The term phlegm refers strictly to secretions from the lungs and tracheobronchial tree. REF: pg. 35 OBJ: 1 8. Excessive sputum production is associated with all the following conditions except: a. Allergies. b. Cigarette smoking. c. Airway infection. d. Pleural infection. ANSWER: D Excessive sputum production is caused most often by inflammation of the mucous glands that line the tracheobronchial tree. Inflammation of these glands occurs most often with airway infection, cigarette smoking, and allergies. REF: pg. 35 OBJ: 1 9. Which of the following terms best describes foul-smelling sputum? a. Fetid b. Mucoid c. Purulent d. Mucopurulent ANSWER: A Copious, foul-smelling (fetid) sputum that separates into layers when standing occurs with bronchiectasis and lung abscess when the patient’s position is changed. REF: pg. 36 OBJ: 1 10. A patient presents in the emergency department with blood-tinged sputum. The term associated with such sputum is: a. Gelatinous. b. Hemoptysis. c. Hematemesis. d. Mucopurulent. ANSWER: B Hemoptysis, expectoration of sputum that contains blood, varies in severity from slight streaking to frank bleeding. REF: pg. 36 OBJ: 1 11. Which of the following conditions is believed to be the most common cause of hemoptysis? a. Tuberculosis b. Erosive bronchitis c. Bronchogenic carcinoma d. Pneumonia ANSWER: B
  • 25. TO GET ALL CHAPTERS EMAIL ME AT>>>>> donc8246@gmail.com Erosive bronchitis in smokers with chronic bronchitis now accounts for nearly half the cases of hemoptysis. REF: pg. 36 OBJ: 1 12. Which of the following definitions is consistent with massive hemoptysis? a. 400 mL in 3 hours b. 400 mL in 12 hours c. 600 mL in 48 hours d. 600 mL in 72 hours ANSWER: A Massive hemoptysis (400 mL in 3 hours, or more than 600 mL in 24 hours) is seen with lung cancers, tuberculosis, bronchiectasis, and trauma. REF: pg. 37 OBJ: 1 13. Hemoptysis in a patient with sudden onset of chest pain who is at risk for venostasis is suggestive of which of the following conditions? a. Pneumonia b. Pulmonary embolism c. Myocardial infection d. Atelectasis ANSWER: B Hemoptysis associated with sudden onset of chest pain and dyspnea in a patient at risk for venous stasis of the legs must lead to assessment of that patient for pulmonary embolism and possible infarction. REF: pg. 37 OBJ: 1 14. In the presence of nausea and vomiting, a history of cirrhosis of the liver suggests which of the following organs as the source of hematemesis? a. Kidneys b. Duodenum c. Esophagus d. Liver ANSWER: C The presence of symptoms such as nausea and vomiting, especially with a history of alcoholism or cirrhosis of the liver, may suggest the esophagus or stomach as the source of hematemesis. REF: pg. 37 OBJ: 1 15. Dyspnea is defined as: a. A visible increase in the work of breathing. b. An increase in respiratory rate and depth. c. Breathlessness as perceived by the patient. d. Difficulty breathing in the upright position. ANSWER: C
  • 26. TO GET ALL CHAPTERS EMAIL ME AT>>>>> donc8246@gmail.com Dyspnea (dys, difficult; pnea, breathing) is defined as a subjective experience of breathing discomfort that consists of qualitatively distinct sensations that vary in intensity. REF: pg. 38 OBJ: 1 16. Which of the following grading systems is useful in qualifying the degree of dyspnea? a. Murray lung injury score b. Modified Borg scale c. Dubowitz score d. Asthma score ANSWER: B The modified Borg scale, as shown as Table 3-5, pg. 39, uses a 0 to 10 grading system, with descriptive terms to depict the perceived intensity of a symptom such as dyspnea after a specified task. REF: Table 3-5, pg. 39 OBJ: 1 17. Dyspnea tends to occur when which of the following is present? a. Work of breathing that is adequate for a given level of exertion b. Decrease in ventilatory drive to breathe c. Increase in ventilatory capacity d. Increase in ventilatory drive to breathe ANSWER: D It is helpful to remember that patients with respiratory disorders will complain of dyspnea when any of the following is present alone or in combination: 1. The work of breathing is abnormally high for the given level of exertion. This is common with narrowed airways, as in asthma, and when the lung is stiff, as in pneumonia. 2. The ventilatory capacity is reduced. This is common when the vital capacity is abnormally low, as is seen in patients with neuromuscular disease. 3. The drive to breathe is elevated beyond normal (e.g., hypoxemia, acidosis, exercise). REF: pg. 39 OBJ: 1 18. Breathing at a rate and depth in excess of the body’s metabolic need is known as: a. Hyperventilating. b. Hyperpnea. c. Platypnea. d. Psychogenic dyspnea. ANSWER: A Hyperventilating is breathing at a rate and depth in excess of the body’s metabolic need, which causes a decrease in arterial carbon dioxide (PaCO2) and results in a decrease in cerebral blood flow. REF: pg. 40 OBJ: 1 19. All of the following conditions are associated with acute dyspnea in children except: a. Asthma. b. Cystic fibrosis.
  • 27. TO GET ALL CHAPTERS EMAIL ME AT>>>>> donc8246@gmail.com c. Bronchiolitis.
  • 28. TO GET ALL CHAPTERS EMAIL ME AT>>>>> donc8246@gmail.com d. Croup. ANSWER: B Acute dyspnea in children is associated most frequently with asthma, bronchiolitis, croup, and epiglottitis. REF: pg. 41 OBJ: 1 20. Which of the following diseases is one of the most common causes of chronic dyspnea in adults? a. Asthma b. Bronchiolitis c. Cystic fibrosis d. Congestive heart failure (CHF) ANSWER: D COPD and chronic CHF are the most common causes of chronic dyspnea in adults. REF: pg. 41 OBJ: 1 21. Paroxysmal nocturnal dyspnea is associated commonly with which of the following conditions? a. Asthma b. COPD c. CHF d. Obstructive sleep apnea ANSWER: C Paroxysmal nocturnal dyspnea (PND) is the sudden onset of difficulty in breathing that occurs when a sleeping patient is in the recumbent position. It often is associated with coughing and is relieved when the patient assumes an upright position. In patients with CHF, PND usually occurs 1 to 2 hours after lying down and is caused by the gradual transfer to the lungs of fluid in the lower extremities. REF: pg. 41 OBJ: 1 22. An inability to breathe while lying down is known as: a. Dyspnea. b. Platypnea. c. Orthopnea. d. PND. ANSWER: C Orthopnea is the inability to breathe when lying down. REF: pg. 41 OBJ: 1 23. Chest pain is the cardinal symptom of which of the following diseases? a. Heart disease b. Pleural disease c. Chest wall disease d. Neuromuscular disease
  • 29. TO GET ALL CHAPTERS EMAIL ME AT>>>>> donc8246@gmail.com ANSWER: A Chest pain is the cardinal symptom of heart disease. REF: pg. 42 | pg. 44 OBJ: 1 24. What is the difference between pleuritic and nonpleuritic chest pain? a. Pleuritic pain is inspiratory; nonpleuritic pain is expiratory. b. Pleuritic pain is sharp and stabbing; nonpleuritic pain is dull and crushing. c. Pleuritic pain usually involves the pleura; nonpleuritic pain involves the lungs. d. Pleuritic pain involves the lung itself; nonpleuritic pain involves the chest wall. ANSWER: B Pleuritic pain, often described as inspiratory pain, is the most common symptom of disease; it is the result of inflammation of the pleura (pleurisy). It is sharp, often abrupt in onset, and severe enough to cause the patient to seek medical help (often within hours of onset). It increases with inspiration, a cough, a sneeze, a hiccup, or laughing. Pleuritic pain usually is localized to one side of the chest—frequently, the lower lateral aspect. It may be only partially relieved by splinting and pain medication. Pleuritic pain increases with pressure and movement but not to the same degree as pain originating from the outer chest wall. In contrast, the lung parenchyma and the visceral pleura that cover the lungs are relatively insensitive to pain; therefore, pain with breathing usually indicates involvement of the parietal pleura. REF: pg. 44 OBJ: 1 25. All the following pulmonary conditions are associated with syncope except: a. Pulmonary embolism. b. Prolonged bouts of coughing. c. Hyperoxia. d. The Valsalva maneuver. ANSWER: C Pulmonary causes of syncope include pulmonary embolism (obstruction of blood flow from the right heart to the left heart), prolonged bouts of coughing (tussive syncope), and hypoxia (low levels of blood oxygen) or hypocapnia (low levels of carbon dioxide). Holding one’s breath following a deep inspiration (Valsalva maneuver) results in high intrathoracic pressure and decreased venous return to the heart. REF: pg. 47 OBJ: 1 26. Cough syncope is associated most often with which of the following types of patients? a. Middle-aged men with underlying COPD b. Pregnant women c. Children with asthma d. Older obese men with diabetes ANSWER: A Cough (tussive) syncope is the transient loss of consciousness that may follow severe coughing. It occurs most commonly in middle-aged men with underlying COPD who are outgoing and moderately obese and have a large appetite for food, alcohol, and smoking.
  • 30. TO GET ALL CHAPTERS EMAIL ME AT>>>>> donc8246@gmail.com REF: pg. 47 OBJ: 1 27. The presence of anasarca is associated commonly with which of the following conditions? a. Dyspnea b. Edema c. Hepatomegaly d. Pleural effusion ANSWER: B Edema is soft tissue swelling that results from an abnormal accumulation of fluid. It may be generalized (anasarca), may appear only in dependent body areas (feet and ankles in ambulatory patients; sacral area in patients restricted to bed rest), or may be limited to a single extremity or organ (such as pulmonary edema). REF: pg. 48 OBJ: 1 28. Which of the following conditions is often associated with right heart failure? a. Organomegaly b. Hepatomegaly c. Loud A2 heart sound d. Shallow breathing ANSWER: B As right heart failure worsens, dependent edema is no longer relieved by rest or changing position, and edema occurs in the abdominal organs, as well as the extremities and dependent areas of the body. As the liver becomes enlarged (hepatomegaly) as a result of edema, the patient may complain of pain just below the ribs on the right side (right upper quadrant pain). REF: pg. 48 OBJ: 1 29. What is the most common manifestation of infection in a patient with pulmonary disease? a. Fever b. Cough c. Diaphoresis d. Increased sputum production ANSWER: A Fever is the most common manifestation of infection. It usually is assumed to be caused by an infectious process until proven otherwise. REF: pg. 49 OBJ: 1 30. The presence of early morning headache may be caused by which of the following conditions? a. Hypoglycemia b. Anoxia c. Hypoxemia d. Hypercapnia ANSWER: D
  • 31. TO GET ALL CHAPTERS EMAIL ME AT>>>>> donc8246@gmail.com Because hypercapnia worsens during sleep in patients with pulmonary disorders, early morning headaches may be the first indication that the patient is retaining abnormally high amounts of carbon dioxide. If hypercapnia persists, headaches may be present throughout the day. REF: pg. 51 OBJ: 1 31. The peak incidence of snoring in adult males occurs at ages years. a. 60 to 64 b. 50 to 59 c. 51 to 55 d. 45 to 50 ANSWER: B The peak incidence of snoring occurs at ages 50 to 59 years in males and at ages 60 to 64 years in women. REF: pg. 52 OBJ: 1 32. Gastroesophageal reflux disease (GERD) is defined as reflux that occurs more than: a. Twice a month. b. Once a month. c. Twice a week. d. Once a week. ANSWER: C Persistent reflux that occurs more than twice a week is often referred to as GERD. REF: pg. 53 OBJ: 1 33. You are a respiratory therapist working in a pediatrician’s office. You see a 16-year-old girl who presents with two chief complaints but otherwise appears to be healthy. The complaints are (1) extremely heavy menstrual bleeding and (2) severe dyspnea following relatively mild exercise. The dyspnea has been getting progressively worse over the past 3 months. Which of the following laboratory studies would you suggest to the physician in order to determine the cause of the dyspnea? a. Pulmonary function studies b. Hemoglobin and hematocrit studies c. Sputum culture and sensitivity studies d. Chest x-ray ANSWER: B Dyspnea during exercise in an otherwise healthy individual may be caused by extreme anemia (hemoglobin <7g/dL). In this case, anemia would likely be associated with the extremely heavy menstrual bleeding. REF: pg. 42 OBJ: 1 34. A sharp, “viselike” chest pain radiating usually down the left arm and possibly spreading to the shoulders, neck, and jaw is called: a. Pleuritic pain. b. Thoracic pain.
  • 32. TO GET ALL CHAPTERS EMAIL ME AT>>>>> donc8246@gmail.com c. Angina. d. Epigastric pain. ANSWER: C Angina is the pain described and is an indication of myocardial ischemia. It often precedes a myocardial infarction. REF: pg. 44 OBJ: 1 35. A patient comes to the emergency department following an episode of unexpected fainting (syncope). In order to determine the potential cause, all of the following could contribute valuable information except: a. A sensation of an irregular heartbeat or history of cardiac arrhythmias. b. The medications taken. c. The activities at the time of the episode. d. The time of day. ANSWER: D It is unlikely that the time of day will have any bearing on the cause of a syncopal episode, especially when factors such as arrhythmias present, medications being taken, and activities being undertaken at the time of the episode are taken into account. REF: pg. 47 OBJ: 1 36. A patient presents at the physician’s office with a complaint of severe, aching pain in her right leg. An examination of the patient’s lower body reveals that the right ankle is badly bruised and severely swollen. The patient states that she fell 2 days ago and “sprained” her ankle but did not seek medical attention at that time. She has been mostly either sitting or lying down since the accident. Her leg pain and swelling could be due to: a. Her sprained ankle. b. Deep vein thrombosis (DVT) in her right leg. c. A side effect from the bruising. d. All of the above. ANSWER: D Any soft tissue, bone, or joint injury in the leg will lead to swelling in the area surrounding the injury. In addition, venous stasis caused by the inactivity as well as bruising can lead to formation of DVT. This is especially dangerous because a deep vein thrombus can break loose and cause a pulmonary embolus, a potentially fatal condition. REF: pg. 48 OBJ: 1 37. A patient is being evaluated for surgery. The therapist hears rales in the lung bases and notes prominent swelling of the ankles. She presses a finger into the edema and notices that a depression remains when she lifts her finger away. The depression is clearly visible for slightly more than 1 minute. This would be classified as pitting edema: a. 1+. b. 2+. c. 3+. d. 4+. ANSWER: C
  • 33. TO GET ALL CHAPTERS EMAIL ME AT>>>>> donc8246@gmail.com Pitting edema that remains visible for 1 to 2 minutes is classified as 3+ pitting edema. REF: Table 3-12, pg. 49 OBJ: 1 38. A patient who appears emaciated presents to the emergency department with a history of low- grade, intermittent, afternoon fevers; frequent, paroxysmal bouts of coughing; hemoptysis; and night sweats. The following should be considered as part of the differential diagnosis (a list of possible disease states that could account for the observed signs and symptoms): a. Tuberculosis b. Pneumonia c. Some types of cancers (e.g., lymphoma) d. All of the above ANSWER: D Choices a, b, and c could result in the observed symptoms. However, the most likely choice is tuberculosis. REF: pg. 36 OBJ: 1 39. Snoring is caused by all of the following except: a. Excessive daytime sleepiness. b. Obesity. c. Excessive alcohol consumption. d. Use of sleeping medications. ANSWER: A Excessive daytime sleepiness is a result of obstructive sleep apnea and is associated with snoring; however, it does not cause snoring. Snoring is due primarily to narrowing of the hypopharynx. This can result from extra tissue in the neck that accompanies obesity, as well as excessive relaxation of the muscles surrounding the hypopharynx caused by alcohol intoxication or sleeping medications. REF: pg. 52 OBJ: 1 40. Gastroesophageal reflux is caused by: a. High activity levels in gastric muscles. b. Blockage of the duodenum. c. Abnormal opening of the pyloric sphincter. d. Abnormal opening of the cardiac (lower esophageal) sphincter. ANSWER: D Inappropriate opening of the cardiac sphincter allows stomach contents to move into the esophagus, resulting in heartburn. Gastric contents reaching the larynx may be aspirated and cause symptoms of gastroesophageal reflux disease (GERD). REF: pg. 53 OBJ: 1
  • 34. TO GET ALL CHAPTERS EMAIL ME AT>>>>> donc8246@gmail.com Chapter4: Vital Signs Test Bank MULTIPLE CHOICE 1. Which of the following is not classified as a classic vital sign? a. Pulse b. Sensorium c. Blood pressure d. Respiratory rate ANSWER: B The four classic vital sign measurements are temperature, pulse, respirations, and blood pressure. REF: pg. 57 OBJ: 1 2. How often should vital signs be recorded in a patient who is hospitalized but is not in intensive care? a. Every hour b. Every 2 hours c. Every 4 to 6 hours d. Every 12 hours ANSWER: C For patients who are hospitalized but are not in intensive care, routine vital signs are recorded most commonly every 4 to 6 hours (at the beginning of a shift and in the middle of a shift). REF: pg. 58 OBJ: 1 3. Which of the following statements regarding vital signs is most accurate? a. Vital signs are used only to establish a baseline. b. Vital signs monitor only the response to therapy. c. Trends in vital signs are far more important than a single measurement. d. Vital signs at baseline should never be compared with those that were obtained 24 hours earlier. ANSWER: C The initial reading generally is referred to as the baseline measurement. A series of vital sign measurements over time establishes a trend and is far more important clinically than any single measurement. Each time vital signs are measured, they should be compared with baseline values and the most recent measurements. Sometimes the patient’s condition may be changing slowly, and comparison with one or two previous measurements does not indicate a trend, whereas comparison over the span of an entire shift or 24-hour period may indicate clearly that the patient is deteriorating slowly. REF: pg. 58 OBJ: 2 4. Which of the following statements best reflects the importance of comparing changes in vital signs?
  • 35. REF: pg. 59 OBJ: 2 TO GET ALL CHAPTERS EMAIL ME AT>>>>> donc8246@gmail.com a. It allows assessment of important subjective patient data. b. It allows recognition of the development of a particular problem. c. It allows detection of additional symptoms. d. It does not allow changes in therapy. ANSWER: B Information about the probability that a patient is experiencing or may be developing a particular problem is obtained by comparing changes in vital signs and other signs and symptoms. REF: pgs. 58-59 OBJ: 2 5. When vital signs are compared with other signs and symptoms to arrive at a conclusion about what is wrong with a patient, this is known as the: a. review of systems. b. differential diagnosis. c. general clinical presentation. d. objective assessment information. ANSWER: B In the field of medicine, this comparison of multiple signs and symptoms to arrive at the patient’s diagnosis is called a differential diagnosis. REF: pg. 59 OBJ: 2 6. If dehydration is suspected in a patient, which of the following parameters should be monitored often? a. Height b. Weight c. Fluid intake and output d. Blood pressure ANSWER: C If a question arises regarding dehydration or fluid overload, fluid intake and output (I & O) and weight may be recorded during each shift until the patient’s fluid balance is stable. REF: pg. 59 OBJ: 2 7. A patient’s general clinical presentation indicates to the respiratory therapist (RT) that the patient is in distress. The RT’s first step should be to: a. call a code and stand by to assist with cardiopulmonary resuscitation (CPR). b. do a complete physical examination to determine the precise cause of the distress. c. evaluate the problem quickly and intervene or locate someone to assist the patient. d. follow the procedure learned in class to introduce himself or herself to the patient and ask the patient what is wrong. ANSWER: C If the patient is in distress, the priorities are to evaluate the problem in the most efficient and
  • 36. REF: pg. 59 OBJ: 3 TO GET ALL CHAPTERS EMAIL ME AT>>>>> donc8246@gmail.com rapid way possible and to intervene or locate someone who can assist the patient.
  • 37. REF: pg. 59 OBJ: 4 TO GET ALL CHAPTERS EMAIL ME AT>>>>> donc8246@gmail.com 8. A well-written description of an RT’s initial observations is beneficial to other healthcare providers because it: a. shows that the RT has established rapport with the patient. b. helps them know how to plan care and relate to the patient’s needs. c. decreases the length of hospitalization of the patient and keeps costs down. d. allows them to read the information quickly without confusion and misunderstandings. ANSWER: B A written description of these initial observations helps others involved in the patient’s care to know how to plan care and relate to the patient’s needs. REF: pg. 60 OBJ: 2 9. Assessing the patient’s level of consciousness is important because it: a. suggests the types of questions that the therapist should ask. b. evaluates the adequacy of cerebral perfusion and oxygenation. c. indicates whether the patient will take an active role in his or her treatment. d. documents the patient’s mental status at the time of the physical examination. ANSWER: B Adequate cerebral oxygenation must be present for the patient to be conscious, alert, and well oriented. REF: pg. 61 OBJ: 3 10. Which of the following scales is the gold standard for assessing trends in the neurologic function of patients who have suffered head trauma? a. Murray scale b. PRISM c. APACHE d. Glasgow scale ANSWER: D The Glasgow coma scale has become the gold standard for assessing trends in the neurologic function of patients who have been sedated, have received anesthesia, have suffered head trauma, or are near coma. REF: pg. 61 OBJ: 3 11. The normal range for oral body temperature in most people is to ° F. a. 97; 99.5 b. 96.7; 98.5 c. 98.7; 100.5 d. 97.7; 98.5 ANSWER: A Normal body temperature for most persons is approximately 98.6° F (37° C), with a normal
  • 38. REF: pg. 59 OBJ: 5 TO GET ALL CHAPTERS EMAIL ME AT>>>>> donc8246@gmail.com range from 97 to 99.5° F and daily variations of 1° to 2° F.
  • 39. TO GET ALL CHAPTERS EMAIL ME AT>>>>> donc8246@gmail.com 12. Fever is defined as an: a. infection in the body. b. elevation of body temperature above normal. c. increase in the oxygen requirements of the body. d. elevation of body temperature above normal because of disease. ANSWER: D When the body temperature is elevated from disease, this elevation is called fever, and the patient is said to be febrile. REF: pg. 62 OBJ: 3 13. What is the most common cause of fever above 102° F? a. Aspiration pneumonitis b. Infection c. Blood transfusion reaction d. Head injury ANSWER: B Infection is most likely to be the cause of fever when the body temperature exceeds 102° F. REF: pg. 62 OBJ: 3 14. For every elevation in body temperature of 1° C, oxygen consumption and carbon dioxide production increase by approximately %. a. 2 b. 5 c. 10 d. 20 ANSWER: C For every elevation in body temperature of 1° C, oxygen consumption and carbon dioxide production increase by approximately 10%. REF: pg. 62 OBJ: 3 15. Which of the following conditions causes hypothermia? a. Damage to the hypothalamus b. Excessive shivering and vasoconstriction c. Exposure to high environmental temperatures d. Invasion of the body by microorganisms that produce an infection ANSWER: A Hypothermia is not common but can occur in persons with a severe head injury that damages the hypothalamus and in those suffering from exposure to cold environmental temperatures. REF: pg. 62 OBJ: 3 16. Which of the following is not considered a typical site for temperature measurement? a. Oral b. Rectal c. Axillary
  • 40. TO GET ALL CHAPTERS EMAIL ME AT>>>>> donc8246@gmail.com d. Fingertip ANSWER: D Body temperature is measured most often at one of four sites: the mouth, ear, axilla, or rectum. REF: pg. 62 OBJ: 4 17. What site is used to measure body temperature in comatose patients? a. Oral b. Axillary c. Rectal d. Ear ANSWER: C Rectal temperatures may be used for patients who are comatose, in intensive care, or confused. REF: pg. 63 OBJ: 4 18. Which of the following is the method of choice for measuring temperature in neonates? a. Oral b. Rectal c. Axillary d. Tympanic ANSWER: C Axillary is the method of choice for neonates because it approximates their core temperature and avoids injury to the rectal tissues. REF: pg. 63 OBJ: 4 19. Which of the following statements is true regarding oral temperature measurements? a. They are the most convenient to obtain and are most readily accepted by alert adults. b. They are useful in infants. c. They are acceptable in orally intubated patients. d. They are affected by delivery of oxygen by nasal cannula. ANSWER: A Oral temperature measurement remains a common, convenient, and acceptable method for awake adult patients. REF: pg. 63 OBJ: 4 20. What is the main advantage of tympanic thermometry over the classic forms of temperature measurement? a. It is fast, clean, and noninvasive. b. Multiple studies have shown a high correlation with core temperature. c. It is the preferred method of temperature measurement in the pediatric population. d. It uses infrared emissions from the surface of the tympanic membrane and is not in contact with skin or other tissue.
  • 41. TO GET ALL CHAPTERS EMAIL ME AT>>>>> donc8246@gmail.com ANSWER: A This method has the advantage of being fast, clean, and noninvasive and avoids the embarrassment and time delays associated with the classic forms of temperature measurement. REF: pg. 63 OBJ: 4 21. Which of the following reasons has been listed as the number one concern regarding use of the tympanic site for body temperature measurement? a. Speed b. Safety c. Accuracy d. Cost ANSWER: C Concerns have been expressed about its accuracy and use in the hospital setting. REF: pg. 63 OBJ: 4 22. The normal pulse rate range for adults is beats/min. a. 60 to 100 b. 70 to 110 c. 80 to 120 d. 90 to 160 ANSWER: A The normal pulse rate for adults is 60 to 100 beats/min and is regular in rhythm. REF: pg. 64 OBJ: 5 23. A pulse rate above the normal range is the definition of which of the following terms? a. Tachypnea b. Arrhythmia c. Bradycardia d. Tachycardia ANSWER: D A pulse rate exceeding 100 beats/min in an adult is termed tachycardia. REF: pg. 64 OBJ: 6 24. What is an important reason to monitor the heart rate in patients with lung disease? a. A heart rate above or below the normal range is dangerous. b. As the heart rate increases, the tissue’s demand for oxygen decreases. c. The more efficiently the heart is pumping, the lower is the resting pulse rate. d. Tachycardia is a common finding when hypoxemia is present. ANSWER: D When the oxygen content of arterial blood falls below normal, usually from lung disease, the heart tries to compensate by increasing cardiac output to maintain adequate oxygen delivery to the tissues. An increase in cardiac output is attained by an increase in heart rate in most persons. For this reason, it is important to monitor the heart rate in patients with lung disease.
  • 42. TO GET ALL CHAPTERS EMAIL ME AT>>>>> donc8246@gmail.com REF: pg. 64 OBJ: 5 25. Which of the following characteristics should be included in the assessment of a patient’s pulse? a. Age, rate, and strength b. Rate, rhythm, and strength c. Age, rhythm, and status d. Strength, amplitude, and volume ANSWER: B The rhythm and strength of the pulse are evaluated, the pulse rate is evaluated, and then the pulse rate is counted. REF: pg. 64 OBJ: 5 26. Which of the following arteries is most appropriate for measuring the pulse of a patient with very low blood pressure? a. Radial artery b. Popliteal artery c. Brachial artery d. Femoral artery ANSWER: D When the blood pressure is abnormally low, the more centrally located pulses, such as the carotid pulse in the neck and femoral pulses in the groin, can be identified more easily than the peripheral pulse. REF: pg. 64 OBJ: 7 27. Which of the following terms is used to describe a significant decrease in pulse pressure during spontaneous inspiration? a. Pulsus alternans b. Pulsus paradoxus c. Pulsus respiratory d. Pulsus asymmetrical ANSWER: B When the patient’s pulse strength decreases with spontaneous inhalation, this is referred to as pulsus paradoxus. REF: pg. 65 OBJ: 6 28. An alternating succession of strong and weak pulses that usually is not related to respiratory disease is known as pulsus: a. tardus. b. alternans. c. paradoxus. d. asymmetrical. ANSWER: B Pulsus alternans is an alternating succession of strong and weak pulses that usually is not related to respiratory disease.
  • 43. REF: pg. 65 OBJ: 6 29. A respiratory rate of greater than breaths/min is considered abnormal at any age. a. 60 b. 50 c. 40 d. 30 ANSWER: A A respiratory rate of 40 breaths/min is unusual for an adult, and a rate greater than 60 breaths/min is abnormal at any age. REF: pg. 65 OBJ: 3 30. In postoperative patients, the degree of tachypnea typically is related to the degree of: a. anesthetic received during surgery. b. pain medication received. c. pneumonia. d. atelectasis. ANSWER: D Tachypnea in the postoperative patient is common when significant fever develops or when the lungs partially collapse (atelectasis) as a side effect of surgery. Atelectasis causes the lungs to become stiffer than normal, and the patient adopts a breathing pattern that is made up of rapid and shallow breaths, which serves as a compensatory mechanism. The degree of atelectasis determines the degree of tachypnea in such cases. REF: pg. 65 OBJ: 6 31. Which of the following breathing patterns describes Kussmaul breathing? a. Normal b. Shallow c. Fast and shallow d. Fast and deep ANSWER: D In the case of ketoacidosis resulting from uncontrolled diabetes, a fast and deep breathing pattern known as Kussmaul breathing is often evident. REF: pg. 65 OBJ: 6 32. Bradypnea may be caused by which of the following? a. Hypothermia b. Narcotic overdose c. Head injury d. All of the above ANSWER: D A slow respiratory rate, referred to as bradypnea, is uncommon but may occur in patients with head injury or hypothermia, as a side effect of certain medications such as narcotics, and in patients with drug overdose.
  • 44. REF: pg. 65 OBJ: 6 33. Which of the following techniques is not useful in measuring the respiratory rate in patients who are spontaneously breathing and alert? a. Ask the patient to breathe normally. b. Lay the hand you are using to count the patient’s pulse on the patient’s abdomen, and watch abdominal movements. c. Count the rate for 30 seconds and multiply by 2. d. Pretend to be counting the pulse while counting the respiratory rate. ANSWER: A Never ask the patient to “breathe normally” while you are assessing the rate of respiration. When individuals think about their breathing, they often voluntarily change their breathing rate and pattern. REF: pg. 66 OBJ: 8 34. The diastolic blood pressure is a reflection of: a. the pressure exerted during left ventricular relaxation. b. the peak pressure exerted during left ventricular contraction. c. the difference between systolic and diastolic pressures. d. the force exerted against the walls of the arteries as blood flows through them. ANSWER: A Diastolic blood pressure is the force that occurs when the heart is relaxed. REF: pg. 66 OBJ: 8 35. Which of the following is not true about pulse pressure? a. It normally is between 35 and 40 mm Hg. b. It is calculated from the 12-lead electrocardiographic printout. c. It is the difference between the systolic and diastolic pressures. d. The peripheral pulse is difficult to detect when the pulse pressure falls to below 30 mm Hg. ANSWER: B Pulse pressure is the difference between the systolic and diastolic pressures. The normal pulse pressure is 35 to 40 mm Hg. When the pulse pressure is less than 30 mm Hg, the peripheral pulse is difficult to detect. Patients with heart failure and inadequate stroke volume usually have a reduced pulse pressure. REF: pg. 66 OBJ: 7 36. Which of the following organs is not at risk for disease in the presence of systemic hypertension? a. Heart b. Kidneys c. Blood vessels d. Pancreas ANSWER: D
  • 45. Persistent pressures at the level of systemic hypertension are associated with an escalating risk for the development of heart, vascular, and renal diseases. REF: pg. 66 OBJ: 9 37. Which of the following is not a cause of hypotension? a. Left ventricular failure b. Peripheral vasodilation c. Hypovolemia d. Right heart failure ANSWER: D Hypotension may occur as the result of peripheral vasodilation, left ventricular failure, or low blood volume. REF: pg. 67 OBJ: 9 38. Hypotension associated with changes in posture in patients with hypovolemia is known as hypotension. a. persistent b. orthostatic c. hypovolemic d. peristaltic ANSWER: B Changes in posture may produce abrupt changes in arterial blood pressure, especially in the hypovolemic patient. Normally, when the patient moves from the supine to the sitting position, the blood pressure changes very little, but when hypovolemia or vasodilation is present, the blood pressure may fall significantly; this is referred to as postural (or orthostatic) hypotension. REF: pg. 67 OBJ: 9 39. When the blood pressure is measured, the sounds that are heard through the stethoscope between the systolic and diastolic pressures are known as sounds. a. Meckel b. brachial c. Korotkoff d. succussion ANSWER: C The auscultatory method of pressure measurement uses the stethoscope to listen for the sounds produced by arterial pulse waves (Korotkoff sounds) when blood flow in the artery resumes. As the pressure is reduced during deflation of the occluding cuff, the Korotkoff sounds change in quality and intensity. REF: pg. 68 OBJ: 8 40. What is the most probable cause for the significant decrease in blood pressure that occurs during inhalation in some patients? a. Hypovolemia b. Variable systemic vascular resistance
  • 46. c. Early left ventricular failure caused by coronary artery insufficiency d. Negative intrathoracic pressure that diminishes blood flow out of the left ventricle ANSWER: D This decrease in systolic blood pressure is more significant during a forced maximal inhalation. When the systolic pressure drops by more than 10 mm Hg during inhalation at rest, a definite abnormality exists; this is termed paradoxic pulse. Paradoxic pulse, also called pulsus paradoxus, occurs in various circulatory and respiratory conditions such as asthma and cardiac tamponade. The most probable mechanism responsible for this fluctuation in blood pressure centers around the negative intrathoracic pressure created by the respiratory muscles during inhalation. REF: pg. 70 OBJ: 11 41. Which of the following conditions is not associated with the presence of pulsus paradoxus? a. Hypervolemia b. Cardiac tamponade c. Constrictive pericarditis d. Asthma ANSWER: A Pulsus paradoxus is commonly seen in patients with restrictions around the heart, such as cardiac tamponade, constrictive pericarditis, or restrictive cardiomyopathy. It also may occur in patients with severe pulmonary diseases, such as acute asthma. REF: pg. 70 OBJ: 11 42. A 42-year-old male patient with a history of poorly controlled asthma is seen in the emergency department by the respiratory therapist (RT). The patient is seated in a chair and clearly is in extreme respiratory distress, with inspiratory and expiratory wheezing throughout. He reports that he has had a fever for the past 2 days and has eaten and drunk very little. He also reports some chest pain, and his blood pressure is 95/55 mm Hg. At one point, the patient stands to move from the chair to the stretcher and has an episode of syncope. The differential diagnosis for this patient should include all of the following except: a. Myocardial infarction (MI). b. Acute asthma exacerbation. c. Dehydration. d. Cerebrovascular accident (CVA; stroke). ANSWER: D The chest pain, although minor, requires evaluation for MI. The wheezing and respiratory distress strongly suggest asthma exacerbation, and the combination of syncope, restricted fluid restriction, and low blood pressure strongly suggest hypovolemia. No signs or symptoms of CVA are present. REF: pg. 70 OBJ: 9 43. An RT attempts to take the blood pressure of the patient in question 42. The RT notes that the pulse intensity determined from palpation of the radial artery decreases during inspiration. The most likely explanation for this would be: a. an (MI).
  • 47. b. an acute asthma exacerbation. c. dehydration. d. a cerebrovascular accident (CVA; stroke). ANSWER: B Because of the high airway resistance caused by the asthma exacerbation, the intrathoracic pressure decreases abnormally during patient inspiration. This results in decreased cardiac filling, decreased blood pressure, and decreased pulse intensity during inspiration. This is an example of pulsus paradoxus. REF: pg. 70 OBJ: 11 44. An RT is called to the emergency department on New Year ’s Eve to assist in the treatment of a 5-year-old child pulled from under the ice of a local pond. When the child arrives, he has been intubated and has a weak, irregularly irregular pulse of 55 beats/min, no spontaneous respiratory effort, and a body temperature of 30o C. The normal practice in this hospital is to use heat–moisture exchangers (HMEs) for all mechanically ventilated patients. Which of the following should the therapist suggest for the ventilator setup? a. Greater than normal rate and tidal volume b. Higher than usual inspiratory flow rate c. Heated humidity in place of the HME d. All of the above ANSWER: C This patient is clearly hypothermic; therefore, along with other warming measures, heated, humidified air will significantly aid in raising the patient’s body temperature. Normally, ventilator settings for these patients require lower minute ventilation than usual. An increased rate and tidal volume would result in increased minute ventilation, whereas an increased inspiratory flow rate would result in an elevated peak inspiratory pressure. REF: pg. 62 OBJ: 3 45. An RT has just finished administering an aerosol treatment by a small-volume nebulizer to a patient who has a serious case of pneumonia. For the past 24 hours, the patient has had a fever of 39 to 40o C. Just as the therapist is finishing, a nursing aide comes in to record the vital signs. She uses an electronic thermometer to record an oral temperature of 36.5o C and comments to the patient, “Well, Mr. Jones, you must be getting better because your fever is completely gone.” The therapist should: a. also verbally encourage the patient by saying that he is getting better. b. totally ignore the situation and continue with the posttreatment assessment. c. complete the treatment and assessment, record the results in the patient’s chart, then go on to the next patient. d. go to the nursing aide after she leaves the room and tell her that the temperature she recorded is probably not accurate and should be repeated. ANSWER: D A small-volume nebulizer produces an aerosol mist that may temporarily cool the inside of the mouth. Hence, an oral temperature should not be measured for at least 10 to 15 minutes following the completion of the nebulizer treatment. REF: pg. 63 OBJ: 3
  • 48. 46. An RT is taking the blood pressure in the arm of a 12-year-old girl. The only sphygmomanometer available has a cuff in which the ends overlap slightly when it is inflated. The pressure that the RT records: a. should be accurate if the measurement is otherwise done properly. b. is likely to be in error, showing a value that is too low. c. is likely to be in error, showing a value that is too high. d. will have a value that cannot be predicted to be either too high or too low. ANSWER: A When measuring the blood pressure in a child, it is permissible to use a cuff where the ends overlap slightly. REF: pg. 68 OBJ: 10
  • 49. Chapter5: Fundamentals of Physical Examination Test Bank MULTIPLE CHOICE 1. All of the following are components of the physical examination except: a. inspection. b. auscultation. c. manipulation. d. percussion. ANSWER: C The four components of the physical examination are inspection, palpation, percussion, and auscultation. REF: pg. 75 OBJ: 1 2. A review of the patient’s history of present illness and past clinical history prior to a physical examination is helpful in that it: a. gives insight into the expected findings during the examination. b. suggests examination techniques to emphasize. c. helps establish a rapport with the patient. d. all of the above. ANSWER: D A review of the history helps direct the course of the physical examination, allows the medical care team to begin to establish rapport with the patient, and may suggest the success or failure of past treatments. REF: pg. 76 OBJ: 2 3. What problem is associated with cyanosis of the oral mucosa? a. Low cardiac output b. Reduced arterial oxygenation c. Hyperventilation d. Hypoventilation ANSWER: B Cyanosis of the oral mucosa is a sign of central cyanosis that occurs only with severe hypoxemia. REF: pg. 76 OBJ: 3 4. What problem is associated with nasal flaring? a. Hypoxemia b. Increased lung compliance c. Increased work of breathing d. Increased dead space ventilation ANSWER: C
  • 50. Nasal flaring most often occurs in infants and children who are working very hard to breathe. Nasal flaring usually occurs when the lungs stiffen (e.g., with pneumonia) or when the airways narrow (e.g., with asthma). REF: pg. 76 OBJ: 3 5. Which of the following is not associated with dilated and unresponsive pupils? a. Opiates b. Atropine c. Brain death d. Catecholamines ANSWER: A Atropine, brain hypoxia, and catecholamines can cause the pupils to become fixed and unresponsive to light. Opiates have no such effect. REF: pg. 76 OBJ: 3 6. Which of the following terms is used to describe constriction of the pupil of the eye? a. Lytosis b. Miosis c. Mydriasis d. Nystagmus ANSWER: B Miosis refers to constriction of the pupils. REF: pg. 76 OBJ: 3 7. Which of the following terms is used to describe drooping of the eyelids? a. Ptosis b. Miosis c. Diplopia d. Chondrosis ANSWER: A Ptosis is defined as drooping of the eyelids. REF: pg. 77 OBJ: 3 8. Which of the following physical examination findings is associated with myasthenia gravis? a. Ptosis b. Miosis c. Mydriasis d. Hepatomegaly ANSWER: A Patients with myasthenia gravis often develop ptosis as part of the clinical picture. REF: pg. 77 OBJ: 3 9. Which of the following terms is used to describe double vision?
  • 51. a. Miosis b. Diplopia c. Nystagmus d. None of the above ANSWER: B Diplopia refers to double vision. REF: pg. 77 OBJ: 3 10. Which of the following is most likely to cause tracheal deviation? a. Left upper lobe pneumonia b. Right upper lobe pneumothorax c. Left lower lobe pleural effusion d. Right lower lobe pleural effusion ANSWER: B Upper lobe changes are more likely to influence tracheal position than are lower lobe problems. Pneumothorax usually causes a volume change in the upper lobe and alters the position of the trachea. REF: pg. 77 OBJ: 3 11. Which of the following is the most common cause of jugular venous distention? a. Auto-positive end-expiratory pressure (auto-PEEP) b. Pneumonia c. Pneumothorax d. Right heart failure ANSWER: D Right heart failure causes a backup of venous blood into the venous system. This causes the neck veins to distend. REF: pg. 78 OBJ: 3 12. At what angle should the head of the bed be elevated to assess jugular vein pressure (JVP)? a. 15 degrees b. 30 degrees c. 45 degrees d. 90 degrees ANSWER: C The current standardized procedure for checking the JVP at the bedside is to elevate the head of the bed to 45 degrees. If the head of the bed were placed at 90 degrees, gravity would make it difficult to see the JVP. If the head of the bed were placed at less than 45 degrees, venous blood would tend to pool in the neck, which would cause most patients to appear to have JVP. REF: pg. 77 OBJ: 4 13. In a patient’s chart, the physician has documented finding tender lymph nodes in the neck region during the physical examination. This problem is consistent with: a. respiratory infection.
  • 52. b. human immunodeficiency virus (HIV) infection. c. lymphoma. d. lung cancer. ANSWER: A Tender lymph nodes often are associated with infection. REF: pg. 78 OBJ: None 14. The horizontal fissure that separates the right upper lobe from the right middle lobe begins at what rib in the midsternal line? a. Rib 2 b. Rib 4 c. Rib 6 d. Rib 8 ANSWER: B The horizontal fissure is located at the fourth rib at the midsternal line. REF: pg. 79 OBJ: 5 15. At what point on the posterior chest wall can the inferior border of the lung normally be found? a. Between T4 and T6 b. Between T6 and T8 c. Between T9 and T12 d. None of the above ANSWER: C Normally, the inferior border of the lung is located between T9 and T12 on the posterior chest. REF: pg. 80 OBJ: 5 16. Which of the following terms describes an abnormal lateral curvature of the spine? a. Kyphosis b. Scoliosis c. Anhidrosis d. Myelosis ANSWER: B The term for lateral curvature of the spine is scoliosis. Kyphosis refers to an abnormal anteroposterior curvature of the spine. REF: pg. 80 OBJ: 6 17. Which of the following disorders is associated with a barrel chest? a. Chronic bronchitis b. Pneumonia c. Emphysema d. Chest trauma ANS: C
  • 53. Patients with emphysema lose their lung recoil. This allows the ribs to straighten somewhat, causing the patient’s anteroposterior diameter to increase significantly. REF: pg. 80 OBJ: 6 18. Which of the following patterns of breathing is associated with a loss in lung volume? a. Slow and deep b. Rapid and deep c. Slow and shallow d. Rapid and shallow ANSWER: D A loss of lung volume causes lung compliance to decrease. This makes it more difficult for the patient to breathe at normal tidal volumes. As a result, patients breathe with smaller volumes but at a faster rate. REF: pg. 80 OBJ: 8 19. What disease is most likely to cause a prolonged expiratory time? a. Asthma b. Pneumonia c. Pneumothorax d. Pulmonary fibrosis ANSWER: A Asthma causes the intrathoracic airways to become narrow, which makes exhalation slow and prolonged. REF: pg. 80 OBJ: 8 20. Which of the following diseases is most likely to cause a prolonged inspiratory time? a. Chronic obstructive pulmonary disease (COPD) b. Asthma c. Epiglottitis d. Acute bronchitis ANSWER: C Epiglottitis causes narrowing of the upper airway. This causes a prolonged inspiratory time because the narrowed upper airway slows inspiratory flow more than expiratory flow. REF: pg. 82 OBJ: 8 21. Which of the following terms describes the sinking inward of the skin overlying the rib cage with each inspiratory effort? a. Ataxia b. Bulging c. Retractions d. Mydriasis ANS: C